Exam 3 study

Lakukan tugas rumah & ujian kamu dengan baik sekarang menggunakan Quizwiz!

A patient has hypokalemia with stable cardiac function. What are the priority nursing interventions? (Select all that apply.) 1. Fall prevention interventions 2. Teaching regarding sodium restriction 3. Encouraging increased fluid intake 4. Monitoring for constipation 5. Explaining how to take daily weights

1. Fall prevention interventions 4. Monitoring for constipation Hypokalemia causes bilateral skeletal muscle weakness, especially in the quadriceps, which creates a risk for falling. Hypokalemia also causes GI smooth muscle weakness, which produces constipation

A family is facing job loss of the father, who is the major wage earner, and relocation to a new city where there is a new job. The children will have to switch schools, and his wife will have to resign from the job she enjoys. Which of the following contribute to this family's hardiness? (Select all that apply.) 1. Family meetings 2. Established family roles 3. New neighborhood 4. Willingness to change in time of stress 5. Passive orientation to life

1. Family meetings 2. Established family roles 4. Willingness to change in time of stress Family hardiness refers to the internal strengths and durability of the family unit. A sense of control over the outcome of life, a view of change as beneficial and growth producing, communication with family members (as in family meetings), well-established family roles among family members, and an active rather than passive orientation in adapting to stressful events all contribute to family hardiness.

The nurse is caring for a patient with a chest tube for treatment of a right pneumothorax. Which assessment finding necessitates immediate notification of the health care provider? 1. New, vigorous bubbling in the water-seal chamber. 2. Scant amount of sanguineous drainage noted on the dressing. 3. Clear but slightly diminished breath sounds on the right side of the chest. 4. Pain score of 2 one hour after the administration of the prescribed analgesic

1. New, vigorous bubbling in the water-seal chamber. The bubbling in the water-seal chamber can mean a new pneumothorax or tube dislodgment. The drainage could be related to the insertion procedure and is scant, so it does not require an immediate phone call to the provider. Answers 3 and 4 are expected findings for a patient with a chest tube.

Which explanation provided by the nurse is the most accurate meaning for "providing culturally congruent care"? 1. It fits the patient's valued life patterns and set of meanings. 2. It is the same set of values as those of the health care team member providing daily care. 3. It holds one's own way of life as superior to those of others. 4. It redirects the patient to a more socially expected set of values.

1. It fits the patient's valued life patterns and set of meanings. The ongoing process of cultural competence refers to the need of the health care professional to be culturally sensitive in providing culturally appropriate care specific to the needs of the patient, family, and community

What should the nurse teach a young woman with a history of UTIs about UTI prevention? (Select all that apply.) 1. Maintain regular bowel elimination. 2. Limit water intake to 1 to 2 glasses a day. 3. Wear cotton underwear. 4. Cleanse the perineum from front to back. 5. Practice pelvic muscle exercises (Kegel) daily

1. Maintain regular bowel elimination. 3. Wear cotton underwear. 4. Cleanse the perineum from front to back. Maintaining regular bowel elimination prevents the rectum from filling with stool, which can irritate the bladder. Adequate hydration will ensure that the bladder is regularly flushed and will help prevent a UTI. Cotton undergarments are recommended. Pelvic muscle exercises promote pelvic health but do not necessarily prevent UTIs.

A 35-year-old woman has Medicaid coverage for herself and two young children. She missed an appointment at the local health clinic to get an annual mammogram because she has no transportation. She gets the annual screening because her mother had breast cancer. Which of the following are social determinants of this woman's health? (Select all that apply.) 1. Medicaid insurance 2. Annual screening 3. Mother's history of breast cancer 4. Lack of transportation 5. Woman's age

1. Medicaid insurance 4. Lack of transportation 5. Woman's age Social determinants of health are the conditions under which people are born, grow, live, work, and age. The social determinants of health are mostly responsible for health disparities. Examples include age, race and ethnicity, socioeconomic status (as reflected by the woman's insurance), access to nutritious food, transportation resources, religion, sexual orientation, level of education, literacy level, disability (physical and cognitive), and geographic location (e.g., access to health care).

The nurse caring for a refugee community identifies that the children are undervaccinated and the community is unaware of resources. The nurse assesses the community and determines that there is a health clinic within a 5-mile radius. The nurse meets with the community leaders and explains the need for immunizations, the location of the clinic, and the process of accessing health care resources. Which of the following practices is the nurse providing? (Select all that apply.) 1. Raising awareness about community resources for the children 2. Teaching the community about health promotion and illness prevention 3. Promoting autonomy in decision making about health practices 4. Improving the health care of the community's children 5. Participating in professional development activities to maintain nursing competency

1. Raising awareness about community resources for the children 2. Teaching the community about health promotion and illness prevention 4. Improving the health care of the community's children Raising awareness about community resources for the children will help the community identify potential clinics for vaccination and well-baby and child examinations. By teaching the community about relevant illnesses, the nurse increases the level of awareness not only about the disease but also about methods of treatment. As the community becomes more informed about the illness, prevention, and treatment methods, the health of the community will increase.

The nurse is caring for a patient with pneumonia. On entering the room, the nurse finds the patient lying in bed, coughing, and unable to clear secretions. What should the nurse do first? 1. Start oxygen at 2 L/min via nasal cannula. 2. Elevate the head of the bed to 45 degrees. 3. Encourage the patient to use the incentive spirometer. 4. Notify the health care provider.

2. Elevate the head of the bed to 45 degrees. The HOB (head of bed) needs to be elevated to help increase lung expansion and ease work of breathing. Also, this makes it easier for the patient to expectorate.

When delegating I&O measurement to assistive personnel, the nurse instructs them to record what information for ice chips? 1. Two-thirds of the volume 2. One-half of the volume 3. One-quarter of the volume 4. Two times the volume

2. One-half of the volume When ice chips melt, their water volume is one-half the volume of the ice chips. The water volume should be recorded as intake.

The nurse is performing tracheostomy care on a patient. What finding would indicate that the tracheostomy tube has become dislodged? 1. Clear breath sounds 2. Patient speaking to nurse 3. SpO2 reading of 96% 4. Respiratory rate of 18 breaths/minute

2. Patient speaking to nurse Patient phonation is a sign that the TT is not in its proper place. All the other findings are normal assessment findings. Refer to Skill 41.2.

A nurse is caring for a 40-year-old man who was diagnosed with Crohn's disease several years ago, resulting in numerous hospitalizations each year for the past 3 years. Which of the following behaviors interfere with the developmental tasks of middle adulthood? (Select all that apply.) 1. Sends birthday cards to friends and family 2. Refuses visitors while hospitalized 3. Self-absorbed in physical and psychological issues 4. Performs self-care activities 5. Communicates feelings of inadequacy

2. Refuses visitors while hospitalized 3. Self-absorbed in physical and psychological issues 5. Communicates feelings of inadequacy Developmental tasks of adulthood can be affected by chronic illness. Self-absorption and the refusal to stay connected with others are of concern to the nurse, as are verbalizations of inadequacy. Staying in touch with friends and performing self-care behaviors demonstrate developmental mastery of adulthood.

Which of the following assessment findings support the nursing diagnosis of an altered self-concept? (Select all that apply.) 1. Uneven gait 2. Slumped posture 3. Avoidance of eye contact 4. Frequent use of the call light 5. Poor personal hygiene

2. Slumped posture 3. Avoidance of eye contact 5. Poor personal hygiene Common assessment findings for an individual with altered self-concept can mirror depressive symptoms,such as slumped posture, poor hygiene, and avoiding intermittent eye contact. An individual with an unsteady or uneven gait may have successfully adjusted to an underlying condition; this does not automatically signal an altered self-concept. Requests for spiritual support and nursing care should be honored and are not related to an altered self-concept

A patient suddenly experienced a severe headache with numbness and decreased movement in the left arm. An emergency brain scan confirmed a cerebral vessel clot. With a stroke confirmed, the emergency room physician consults with a neurosurgeon to schedule an emergent angiogram to remove the clot. Which teaching approach is most appropriate for explaining to the patient what to expect from the procedure? 1. Selling approach 2. Telling approach 3. Entrusting approach 4. Participating approach

2. Telling is the best approach when there is limited time for teaching information.

A mother and her two children are homeless and enter a free health care clinic. Which statements most likely describe the effects of homelessness on this family? (Select all that apply.) 1. The children have stability in their education. 2. The family members may have symptoms of malnutrition, such as anemia. 3. The family is at a low risk for experiencing violence. 4. The children are at higher risk for developing ear infections. 5. All family members may have mental health issues

2. The family members may have symptoms of malnutrition, such as anemia. 4. The children are at higher risk for developing ear infections. 5. All family members may have mental health issues Families who are homeless often experience difficulty in accessing food and have a poor nutritional status. Children are at a higher risk for developing ear infections. Homelessness puts all family members at risk for developing mental health issues. Children have difficulty proving residency for school enrollment, so they are more likely to not be in school or to drop out. Families that are homeless are also at a higher risk for experiencing physical and emotional violence.

A nurse who works in an outpatient chemotherapy infusion center is assigned to the care of a 56-year-old male patient who is receiving chemotherapy for colon cancer. This is the patient's first clinic visit. The nurse reviews the patient's medical record and sees a note about the patient receiving instruction on how chemotherapy treats cancer, but the note does not summarize the patient's response. The doctor makes a quick visit and tells the patient, "We will get your treatment started today and we will be checking your blood each week for any problems." The doctor leaves and the patient asks the nurse, "What are the blood tests for?" To determine this patient's learning needs, what should the nurse assess? (Select all that apply.) 1. Medical record summary of the stage of the cancer 2. The patient's behavior as the nurse interacts with him 3. The patient's level of knowledge about chemotherapy effects 4. The number of treatments the patient will be receiving 5. The patient's health literacy 6. The patient's self-description of severity of his cancer 7. The date of the medical record note describing patient instruction

2. The patient's behavior as the nurse interacts with him 3. The patient's level of knowledge about chemotherapy effects 5. The patient's health literacy 6. The patient's self-description of severity of his cancer The nurse assesses the patient's motivation to learn by noting behavior and looking for the patient's eye contact, attention span, and tendency to ask questions. The patient's level of knowledge will determine within the cognitive domain what the patient already knows and how to target future instruction. The patient's health literacy will determine the approaches to use for instruction and selection of any written or visual resources. The patient's self-description of his cancer is important to determine perception of disease, affecting motivation to learn. Assessment of the medical record, number of treatments, and date of record note will not contribute to assessment of learning needs.

While administering medications, a nurse realizes that a prescribed dose of a medication was not given. The nurse acts by completing an incident report and notifying the patient's health care provider. Which of the following is the nurse exercising? 1. Authority 2. Responsibility 3. Accountability 4. Decision making

3. Accountability is nurses being answerable for their actions. It means nurses accept the commitment to provide excellent patient care and the responsibility for the outcomes of the actions in providing that. Following agency policy for reporting medication errors demonstrates the nurse's commitment to safe patient care.

What is a critical step when inserting an indwelling catheter into a male patient? 1. Slowly inflate the catheter balloon with sterile saline. 2. Secure the catheter drainage tubing to the bedsheets. 3. Advance the catheter to the bifurcation of the drainage and balloon ports. 4. Advance the catheter until urine flows, then insert ¼ inch more.

3. Advance the catheter to the bifurcation of the drainage and balloon ports. Advancing the catheter to the bifurcation avoids inflating the catheter balloon in the prostatic urethra, causing trauma and pain. Catheter balloons are never inflated with saline. Securing the catheter drainage tubing to the bedsheets increases the risk for accidental pulling or tension on the catheter. Advancing the catheter until urine flows and then inserting it ¼ inch more is not unique to the male patient.

What factor results in vulnerable populations being more likely to develop health problems? 1. The ability to use available resources to find housing 2. Adequate transportation to the grocery store and community clinics 3. Availability of others to help provide care 4. Limited access to health care services

4. Limited access to health care services It is the excess of risks and combination of risk factors that make this population more vulnerable. These vulnerabilities can be associated with the individual's/community's social determinants of health or individual health disparities

The nurse assesses pain and redness at a VAD site. Which action is taken first? 1. Apply a warm, moist compress. 2. Aspirate the infusing fluid from the VAD. 3. Report the situation to the health care provider. 4. Discontinue the IV infusion

4. Discontinue the IV infusion Pain and redness at a VAD site are indicators of phlebitis. When phlebitis occurs, the infusion must be stopped and the VAD removed as the highest priority

A mother is concerned about her child's flulike symptoms. You learn from the health assessment that the mother practices the use of "hot" and "cold" foods to treat ailments. Which of the following foods do you expect the mother to use to treat her child? 1. Chicken 2. Yogurt 3. Fresh fruits 4. Eggs

4. Eggs Certain cultures believe in the importance of balance and harmony in health. Natural or holistic balance is believed to be achieved by using "hot" and "cold" foods as remedies to treat illness. Eggs are an example of a "hot" treatment.

Which task is appropriate for a registered nurse (RN) to delegate to an AP? 1. Determining whether the patient understands the preoperative preparation required before the surgery in the morning 2. Administering the ordered antibiotic to the patient before surgery 3. Obtaining the patient's signature on the surgical informed consent 4. Helping the patient to the bathroom before leaving for the operating room

4. Helping the patient to the bathroom before leaving for the operating room Assisting the patient with toileting activities is within the scope of an AP's duties. The other activities require the skill and knowledge of the RN.

Place the following steps for insertion of an indwelling catheter in a female patient in appropriate order. 1. Insert and advance catheter. 2. Lubricate catheter. 3. Inflate catheter balloon. 4. Cleanse urethral meatus with antiseptic solution. 5. Drape patient with the sterile square and fenestrated drapes. 6. When urine appears, advance another 2.5 to 5 cm. 7. Prepare sterile field and supplies. 8. Gently pull catheter until resistance is felt. 9. Attach drainage tubing

5. Drape patient with the sterile square and fenestrated drapes. 7. Prepare sterile field and supplies 2. Lubricate catheter. 4. Cleanse urethral meatus with antiseptic solution 1. Insert and advance catheter. 6. When urine appears, advance another 2.5 to 5 cm. 3. Inflate catheter balloon. 8. Gently pull catheter until resistance is felt. 9. Attach drainage tubing

A nurse is preparing to perform endotracheal suctioning for a client. The nurse should follow which of the following guidelines? (select all that apply) A) Apply suction while withdrawing the catheter. B) Perform suctioning on a routine basis every 2 to 3 hrs. C) Maintain medical asepsis during suctioning. D) Use a new catheter for each suctioning attempt. E) Apply suction for 10 to 15 seconds

A) Apply suction while withdrawing the catheter. D) Use a new catheter for each suctioning attempt. E) Apply suction for 10 to 15 seconds

A nurse is caring for a client who has a tracheostomy. Which of the following actions should the nurse take when providing tracheostomy care? (select all that apply) A) Apply the oxygen source loosely if the SpO2 decreases during the procedure. B) Use surgical asepsis to remove and clean the inner cannula. C) Clean the outer cannula surfaces in a circular motion from the stoma site outward D) Replace the tracheostomy ties with new ties E) Cut a slit in the gauze squares to place beneath the tube holder

A) Apply the oxygen source loosely if the SpO2 decreases during the procedure. B) Use surgical asepsis to remove and clean the inner cannula. C) Clean the outer cannula surfaces in a circular motion from the stoma site outward

A nurse is reviewing the medical record of a client who has hypocalcemia. The nurse should identify which of the following findings as a risk factor of the development of this electrolyte imbalance? A) Crohn's disease B) Postoperative following appendectomy C) History of bone cancer D) Hyperthyroidism

A) Crohn's disease: Chron's disease is a risk factor for hypocalcemia. This malabsorption disorder places the client at risk for hypocalcemia due to inadequate calcium absorption.

A nurse educator is presenting a module on basic first aid for newly licensed home health nurses. The client who has heat stroke will have which of the following? A) Hypotension B) Bradycardia C) Clammy skin D) Bradypnea

A) Hypotension is a manifestation of heat stroke

A nurse is planning care for a client who has hypernatremia. Which of the following actions should the nurse include in the plan of care? A) Infuse hypotonic IV fluids B) Implement a fluid restriction C) Increase sodium intake D) Administer sodium polystyrene sulfonate.

A) Infuse hypotonic IV fluids: Hypotonic IV fluids are indicated for the treatment of hypernatremia related to fluid loss to expand the ECF volume and rehydrate the cells.

A nurse is assessing a client who has an acute respiratory infection, increasing the risk for hypoxemia. Which of the following findings are early indications that should alert the nurse that the client is developing hypoxia? (select all that apply.) A) Restlessness B) tachypnea C) Bradycardia D) Confusion E) Hypertension

A) Restlessness B) tachypnea D) Confusion E) Hypertension

A senior student nurse delgates the task of intake and output to a new nursing assistant. The student will verify that the nursing assistant understands the task of I&O when the nursing assistant states, A. " I will record the amount of all voided urine" B. " I will not count liquid stools as output" C. "I will not record a cafe mocha as intake." D. "I will notate perspiration and record it as a small or large amount."

A. " I will record the amount of all voided urine"

1. Jay delegates the task of taking vital signs to a new nursing assistant on the unit with whom Jay has never worked before. Which of the following questions asked of the new nursing assistant will help Jay determine that it is safe to delegate the task of taking vital signs? (Select all that apply.) A. "How long should you keep the thermometer in the patient's mouth?" B. "How do you apply a blood pressure cuff to an obese patient?" C. "How do you take a patient's pulse?" D. "Do you know how to take vital signs?"

A. "How long should you keep the thermometer in the patient's mouth?" B. "How do you apply a blood pressure cuff to an obese patient?" C. "How do you take a patient's pulse?"

2. Mrs. McIntosh displays signs of altered self-concept during the patient interview with Walter. What are the signs of altered self-concept? (Select all that apply.) A. Avoidance of eye contact B. Lack of interest C. Difficulty in making a decision D. Rapid speech E. Unkempt appearance

A. Avoidance of eye contact B. Lack of interest C. Difficulty in making a decision E. Unkempt appearance

3. For Jeffrey to deliver acceptable care to Mrs. Lasky, he must exhibit cultural competence. Which of the following are the interlocking components of cultural competence? (Select all that apply.) A. Cultural awareness B. Cultural imposition C. Cultural knowledge D. Cultural skills E. Cultural encounters

A. Cultural awareness C. Cultural knowledge D. Cultural skills E. Cultural encounters

1. Mr. Smith tells Margaret that he doesn't think he can hold the spoon on his own and feed himself. Mr. Smith lacks self-efficacy. Self-efficacy comes from which of the following sources? (Select all that apply.) A. Enactive mastery experiences B. Vicarious experiences C. Auditory persuasion D. Physiological states

A. Enactive mastery experiences B. Vicarious experiences D. Physiological states

2. Rochelle assesses Mr. Sanders' support system to ensure that he is receiving the attention he requires. The strength of Ms. Jones and Mr. Sanders' relationship and their ability to adapt to challenges and stressors shows that their relationship is built on which of the following factors? (Select all that apply.) A. Hardiness B. Friendship C. Resiliency D. Compatibility E. Trust

A. Hardiness C. Resiliency

4. Franz studies acid-base balance. Which two organs are responsible for acid excretion, which helps maintain acid-base balance? A. Lungs and kidneys B. Kidneys and liver C. Bladder and bowel D. Lungs and bladder

A. Lungs and kidneys

You are caring for a patient. Visitors at the bedside include the patient's life partner, widowed father, brother, and niece. Th nurse acknowledges that current trends in American families include: A. couples without children B. more singles choosing to live alone. C. a very different look from 15 years ago D. a mother, father, and more than one child

C. a very different look from 15 years ago

A patient with long-standing history of diabetes mellitus is voicing concerns about kidney disease. The patient asks the nurse where urine is formed in the kidney. The nurse's response is ther: A.bladder B.kidney C.nephron D.ureter

C.nephron

respiratory acidosis causes

COPD, Pulmonary Embolism, and opioid overdose

You are assigned to care for a patient who retired 6 months ago. While providing care, you identify that this patient is struggling emotionally with change. This situation is most likely associated with the self-concept component of: A. Identity stressor B. Sexuality stressor C. Body image stressor D. Role-performance stressor

D. Role- performance stressor

After evaluating a patient's external variables, the nurse concludes that health beliefs and practices can be influenced by A. Emotional factors B. Intellectual background C. Developmental stage D. Socioeconomic factors

D. Socioeconomic factors

A young girl is having problems urinating postoperatively. You remember that children may have trouble voiding: A. in bathrooms other than their own. B. in a urinal. C. while lying in bed. D. in the presence of a person other than one of their parents.

D. in the presence of a person other than one of their parents.

1. Ms. Ruiz has tuberculosis. Because of this, like other community nurses, Michelle will also serve in the role of _____________ in an effort to promote health teaching about tuberculosis to Ms. Ruiz and track incident rates of the illness.

Epidemiologist

1. Mr. Burke's respiratory rate as determined by Cyrus is 42 breaths/min. This means that he is experiencing apnea. A. True B. False

False

You will use the concept of primary prevention when instructing a patient to A. Get a flu shot every year. B. Take a blood pressure reading every day. C. Explore hiring a patient with a known disability. D. Undergo physical therapy following a cerebrovascular accident.

Get a flu shot every year.

What assessments does a nurse make before hanging an IV fluid that contains potassium? (Select all that apply.) 1. Urine output 2. ABGs 3. Fullness of neck veins 4. Serum potassium laboratory value in EHR 5. Level of consciousness

1. Urine output 4. Serum potassium laboratory value in EHR Increased potassium intake when potassium output is decreased or during hyperkalemia is a major risk for hyperkalemia. Before increasing IV potassium intake, check to see that urine output is normal and that the serum potassium level in the health record is not above normal.

A community health nurse conducts a community assessment focused on adolescent health behaviors. The nurse determines that a large number of adolescents smoke. Designing a smoking-cessation program at the youth community center is an example of which nursing role? 1. Epidemiologist 2. Counselor 3. Collaborator 4. Case manager

2. Counselor To engage a patient to participate in a smoking-cessation program or any program that requires changing a behavior requires the nurse to act as a counselor to support the patient in changing that behavior (e.g., in this case a smoking habit). The nurse does not educate the patient about the dangers of smoking but first must actively counsel the patient to decide to change the behavior. Without support and counseling, the smoking-cessation education may not be used effectively by the patient

respiratory alkalosis causes

hyperventilation (anxiety, PE, fear), mechanical ventilation

metabolic acidosis s/s

kussmaul's respirations N/V lethargy, confusion HA muscle twitching hyperkalemia

1. When interviewing Ms. Ruiz, it is okay to have Mr. Ruiz present as long as he understands that he may not be abusive during the interview. A. True B. False

B. False

3. Franz knows that hypercalcemia and hypermagnesemia increase neuromuscular excitability. A. True B. False

B. False

A married couple has three children. The youngest child just graduated from college and is moving to a different city to take a job. The other two children left home several years ago. The parents of both spouses are older and are beginning to need help to maintain their home. What assessment questions will help the nurse determine the family's functioning? (Select all that apply.) 1. "Which transitions or changes in your family are you currently experiencing?" 2. "Are your children having any problems that are affecting your family right now?" 3. "Describe a recent family conflict and how your family resolved it." 4. "What coping strategies do you typically use as a family?" 5. "Who is involved in helping care for your parents?"

"Describe a recent family conflict and how your family resolved it." "What coping strategies do you typically use as a family?" "Who is involved in helping care for your parents?" This couple no longer has children living at home and has older parents to assist. Assessment questions about family functioning address how individuals behave in relation to one another, which is reflected in questions that determine how the family completes routine activities (e.g., making meals, doing laundry), communicates, and solves problems

A family consisting of a grandparent, two adults, and three school-age children just immigrated to the United States. They come to a community wellness center to establish health care. Which of the following questions does the nurse ask to assess the family's function? (Select all that apply.) 1. "What does your family do to keep members healthy?" 2. "How does your family usually make decisions?" 3. "What health services are available in your neighborhood?" 4. "Which rituals or celebrations are important for your family?" 5. "Is there a lot of crime in your neighborhood?" 6. "How many parks are there in your community?"

"What does your family do to keep members healthy?" "How does your family usually make decisions?" "Which rituals or celebrations are important for your family?" Cultural practices, decision making, and rituals and celebrations describe what is important to a family. This set of assessment data helps you gain insight into how the family functions and what family members consider important to their health and family functioning.

In identifying needs for health policy and health program development and services, a community assessment focuses on which of the following elements? A. Structure B. People C. Social systems D. Environments E. All of the above

E. All of the above

1. Mr. Smith constantly denies that he has dementia by stating, "I'm old. A little forgetfulness is normal." Mr. Smith is in the _________ or _________ stage of grieving.

denial and disbelief

metabolic alkalosis s/s

- restlessness, followed by lethargy - dysrhythmias, tachycardia - *compensatory hypoventilation* - confusion, decreased LOC, dizzy, irritable - nausea, vomiting, diarrhea - *tremors, muscle cramps, tingling of fingers and toes* - *hypokalemia*

respiratory acidosis s/s

-Fatigue -Drowsiness -Dysrhythmia -Dyspnea -Headache -Hyperkalemia -Hypoventilation

Match the cultural concepts on the left with the correct definitions on the right. ____ 1. Etic worldview ____ 2. Worldview ____ 3. Cultural desire ____ 4. Intersectionality ____ 5. Emic worldview a. Factor that shapes how people perceive others and how they relate to reality b. Insider's perspective in an intercultural encounter c. A policy model that describes factors and power structures that shape and influence life d. An outsider's perspective in an intercultural encounter e. The motivation of a health care professional to "want to" engage in cultural competence

1 d. An outsider's perspective in an intercultural encounter 2. a. Factor that shapes how people perceive others and how they relate to reality 3 e. The motivation of a health care professional to "want to" engage in cultural competence 4 c. A policy model that describes factors and power structures that shape and influence life 5 b. Insider's perspective in an intercultural encounter

Which statements made by a nursing student about the teachback technique show understanding of the technique? (Select all that apply.) 1. "After teaching a patient how to use an inhaler, I need to use the teach-back technique to test my patient's ability to use the inhaler correctly." 2. "The teach-back technique is an ongoing process of asking patients for feedback." 3. "Using teach-back will help me identify explanations and communication strategies that my patients will most commonly understand." 4. "Using pictures, drawings, and models can enhance the effectiveness of the teach-back technique." 5. "When doing my patient teaching, I will use plain language to make the material easier to understand for the patient."

1. "After teaching a patient how to use an inhaler, I need to use the teach-back technique to test my patient's ability to use the inhaler correctly." 2. "The teach-back technique is an ongoing process of asking patients for feedback." 3. "Using teach-back will help me identify explanations and communication strategies that my patients will most commonly understand." 4. "Using pictures, drawings, and models can enhance the effectiveness of the teach-back technique." 5. "When doing my patient teaching, I will use plain language to make the material easier to understand for the patient." Teach-back is a method to evaluate patient learning after teaching is completed. Evaluation of learning can take place through verbal questioning or by asking a patient to demonstrate a skill, such as using an inhaler. Using teach-back helps you identify the best communication strategies to use with patients. You use plain language to help the patient better understand what you are teaching.

A nurse is seeing all of these patients in the community health clinic. The nurse identifies which patient(s) as having a health disparity? (Select all that apply.) 1. A patient who has a homosexual sexual preference 2. A patient unable to access primary care services 3. A patient living with chronic schizophrenia 4. A family who relies on public transportation 5. A patient who has a history of hypertension

1. A patient who has a homosexual sexual preference 2. A patient unable to access primary care services 3. A patient living with chronic schizophrenia Sexual preference or gender identity; mental health issues or sensory, cognitive, or physical disabilities; and limited access to health care are identified as health disparities. Health disparities adversely affect certain groups of people and cause obstacles to health. Health disparities can also be related to race, ethnicity, socioeconomic status, religion, age, or geographic location

Which of the following steps are necessary when inserting a shortperipheral IV? (Select all that apply.) 1. Apply tourniquet to arm 10 to 15 cm (4-6 inches) above the intended insertion site. 2. Cleanse skin using an approved antiseptic agent such as alcoholbased 2% chlorhexidine and allow to dry thoroughly. 3. Stabilize the vein by placing the thumb proximal to the insertion site, stretching the skin in the direction of insertion. 4. Use the smallest-gauge, shortest catheter available and insert with the bevel up at a 10- to 15-degree angle. 5. Observe for blood in the flashback chamber of the catheter and advance the catheter off the needle into the vein. 6. Release the tourniquet once the catheter has been secured and the dressing has been applied.

1. Apply tourniquet to arm 10 to 15 cm (4-6 inches) above the intended insertion site. 2. Cleanse skin using an approved antiseptic agent such as alcohol based 2% chlorhexidine and allow to dry thoroughly. 4. Use the smallest-gauge, shortest catheter available and insert with the bevel up at a 10- to 15-degree angle. 5. Observe for blood in the flashback chamber of the catheter and advance the catheter off the needle into the vein. Placing the thumb proximal to the insertion site would not allow for adequate stabilization of the vein and could allow the clinician to be stuck by the needle. The vein should be stabilized by placing the thumb distal to the insertion site and stretching the skin against the direction of insertion. The tourniquet is released once blood flashback has been observed and the catheter has been advanced into the vein. Waiting to release the tourniquet until the catheter has been secured and the dressing applied would promote backflow of blood that can occlude the catheter if the blood clots

A patient is scheduled to have an intravenous pyelogram (IVP) tomorrow morning. Which nursing measures should be implemented before the test? (Select all that apply.) 1. Ask the patient about any allergies and reactions. 2. Instruct the patient that a full bladder is required for the test. 3. Instruct the patient to save all urine in a special container. 4. Ensure that informed consent has been obtained. 5. Instruct the patient that facial flushing can occur when the contrast medium is given.

1. Ask the patient about any allergies and reactions. 4. Ensure that informed consent has been obtained. 5. Instruct the patient that facial flushing can occur when the contrast medium is given. An IVP involves intravenous injection of an iodine based contrast medium. Patients who have had a previous hypersensitivity reaction to contrast media are at high risk for another reaction. Informed consent is required. The patient may experience facial flushing during injection of the contrast medium. There is no need to have a full bladder as with a pelvic ultrasound examination or to save any urine for testing. There is no instrumentation of the urinary tract as with cystoscopy

The public health nurse is working with the county health department on a task force to fully integrate the goals of Healthy People 2030. Most of the immigrant population do not have a primary care provider, nor do they participate in health promotion activities; the unemployment rate in the community is 25%. How does the nurse determine which goals need to be included or updated? (Select all that apply.) 1. Assess the health care resources within the community. 2. Assess the existing health care programs offered by the county health department. 3. Compare existing resources and programs with Healthy People 2030 goals. 4. Initiate new programs to meet Healthy People 2030 goals. 5. Implement educational sessions in the schools to focus on nutritional needs of the children.

1. Assess the health care resources within the community. 2. Assess the existing health care programs offered by the county health department. 3. Compare existing resources and programs with Healthy People 2030 goals.The nurse must first assess for existing health care resources and educational programs. Then the nurse must compare these resources and programs with Healthy People 2030 goals. These processes determine whether any new goals need to be added or updated.

A nurse is caring for a 66-year-old patient who lives alone and is receiving chemotherapy and radiation for a new cancer diagnosis. He is unable to care for himself because of severe pain and fatigue. He moves into his 68-year-old brother's home so his brother can help care for him. Which assessment findings indicate that this family caregiving situation will be successful? (Select all that apply.) 1. Both the patient and his brother attend church together regularly. 2. The brothers are living together and enjoy eating the same foods. 3. Other siblings live in the same city and are willing to help. 4. The patient and his brother have a close network of friends. 5. The patient has obsessive-compulsive disorder and has difficulty throwing away possessions.

1. Both the patient and his brother attend church together regularly. 3. Other siblings live in the same city and are willing to help. 4. The patient and his brother have a close network of friends. Many older adults use their faith and spirituality to cope with life changes. Having a close social network and other family members who are willing to assist with caregiving helps to alleviate the stress experienced by caregivers. Potential risk factors for caregiver distress and burden in this relationship include that the brothers are living together and that one of the brothers has a mental illness

A nurse is working with an older adult who recently moved to an assisted-living center because of declining physical capabilities and socialization. Which nursing interventions are directed at promoting self-esteem in this patient? (Select all that apply.) 1. Commending the patient's efforts at completing self-care tasks 2. Assuming that the patient's physical complaints are attentionseeking measures 3. Encouraging the patient to add personal items to the residence 4. Minimizing time discussing memories and past achievements with the patient 5. Providing decision-making opportunities to join the center's daily activities program

1. Commending the patient's efforts at completing self-care tasks 3. Encouraging the patient to add personal items to the residence 5. Providing decision-making opportunities to join the center's daily activities program Reinforce efforts to complete self-care tasks, allowing additional time to complete tasks. Personalizing the residence provides choice and familiarity to a new area. This patient is an adult and needs opportunities to make care and social decisions. Focusing on the center's daily activities provides decision making and opportunities for more socialization. Each of these interventions maintains or promotes independence, which also fosters self-esteem and confidence. It is important to refrain from making assumptions, as in assuming that physical complaints are attention seeking. Time should be allocated to review past accomplishments and memories. Minimizing discussion of past memories and achievements reduces the importance of these events and in turn may affect self-esteem

A community health nurse is working in a clinic with a focus on asthma and allergies. What is the primary focus of the community health nurse in this clinic setting? (Select all that apply.) 1. Decrease the incidence of asthma attacks in the community 2. Increase patients' ability to self-manage their asthma 3. Treat acute asthma in the hospital 4. Provide asthma education programs for the teachers in the local schools 5. Provide scheduled immunizations to people who come to the clinic

1. Decrease the incidence of asthma attacks in the community 2. Increase patients' ability to self-manage their asthma 4. Provide asthma education programs for the teachers in the local schools All these activities improve the level of health and quality of life for patients in this community. Asthma self-management controls symptoms and improves a patient's quality of life. Assessing for and preventing risks, as well as educational programs, improve the level of health within a community. Managing chronic diseases in the community improves the overall level of health of that community.

A patient has been laid off from his construction job and has many unpaid bills. He is going through a divorce from his marriage of 15 years and has been praying daily to help him through this difficult time. He does not have a primary health care provider because he has never really been sick, and his parents never took him to a physician when he was a child. Which external variables influence the patient's health practices? (Select all that apply.) 1. Difficulty paying his bills 2. Praying daily 3. Age of patient (46 years) 4. Stress from the divorce and the loss of a job 5. Family practice of not routinely seeing a health care provider

1. Difficulty paying his bills 5. Family practice of not routinely seeing a health care provider External factors impacting health practices include family beliefs and economic impact. The way in which patients' families use health care services generally affects their health practices. Their perceptions of the seriousness of diseases and their history of preventive care behaviors (or lack of them) influence how patients think about health. Economic variables may affect a patient's level of health by increasing the risk for disease and influencing how or at what point the patient enters the health care system

A patient who is newly divorced states, "Although I'm really scared about what happens next to my children and myself, I know that divorce is not uncommon today." What term does the nurse use in the patient's medical record to describe the characteristic displayed in this statement? 1. Durability 2. End-of-life care 3. Family functioning 4. Family's culture

1. Durability Family durability is a system of support and structure within a family that extends beyond the walls of the household. For example, marriages may end in divorce or death, and remarriage may occur, or children may leave home as adults, but in the end the "family" transcends long periods and inevitable lifestyle changes

The public health nurse is working with the local city/county health department during a pandemic that has created a crisis within the community. What are responsibilities of the public health nurse during the pandemic? (Select all that apply.) 1. Educate the public on disease prevention 2. Serve as liaison between patients and health care services and providers 3. Investigate cases as they arise 4. Monitor trends of the disease outbreak 5. Assist with testing for identification of the disease

1. Educate the public on disease prevention 2. Serve as liaison between patients and health care services and providers 3. Investigate cases as they arise 4. Monitor trends of the disease outbreak 5. Assist with testing for identification of the disease During a pandemic, public health nurses are key members of the health care team. They can serve as the liaison between the patient and health care team, assisting patients with accessing needed testing and treatment. Besides providing patient care, they are integral to identifying and tracking new cases, along with monitoring trends in disease outbreak. Educating the public about disease prevention and treatment are important responsibilities for the nurse.

A nurse in a community health clinic reviews screening results from students in a local high school during the most recent academic year. The nurse discovers a 10% increase in the number of positive tuberculosis (TB) skin tests when comparing these numbers to the previous year. The nurse contacts the school nurse and the director of the health department. Together they begin to expand their assessment to all students and employees of the school district. The community nurse is acting in which nursing role(s)? (Select all that apply.) 1. Epidemiologist 2. Counselor 3. Collaborator 4. Case manager 5. Caregiver

1. Epidemiologist 3. Collaborator When the community health nurse initially noticed an increase in the number of positive TB skin tests, the nurse was comparing current data with previous data to track positive skin test rates. Once the increase was noted, the community health nurse collaborated with the school nurse and other members of the health department to determine the impact of the increased positive TB skin tests.

The nurse can increase a patient's self-awareness and self-concept through which of the following actions? (Select all that apply.) 1. Helping the patient define personal problems clearly 2. Allowing the patient to openly explore thoughts and feelings 3. Reframing the patient's thoughts and feelings in a more positive way 4. Having family members assume more responsibility during times of stress 5. Recommending self-help reading materials

1. Helping the patient define personal problems clearly 2. Allowing the patient to openly explore thoughts and feelings 3. Reframing the patient's thoughts and feelings in a more positive way Helping a patient define problems clearly, allowing the patient to openly explore thoughts and feelings, and reframing the patient thoughts and feelings in a more positive way are designed to promote self-awareness and a positive self-concept. Having the family assume more responsibility does not help a patient achieve self-awareness; instead, it is important to encourage the patient to assume more self-responsibility. The nurse should refrain from offering self-help reading materials unless directly asked; the nurse should then provide numerous options.

When taking care of patients, a nurse routinely asks whether they take any vitamins or herbal medications, encourages family members to bring in music that the patients like to help the patients relax, and frequently prays with her patients if that is important to them. The nurse is practicing which model? 1. Holistic 2. Health belief 3. Transtheoretical 4. Health promotion

1. Holistic The nurse is using a model of care that takes a more holistic view of health, with emotional and spiritual well-being and other dimensions of an individual considered to be important aspects of physical wellness. The holistic health model of nursing attempts to create conditions that promote optimal health. Nurses using the holistic nursing model recognize the natural healing abilities of the body and incorporate complementary and alternative interventions such as music therapy, reminiscence, relaxation therapy, therapeutic touch, and guided imagery because they are effective, economical, noninvasive, nonpharmacological complements to traditional medical care.

An IV fluid is infusing more slowly than ordered. The infusion pump is set correctly. Which factors could cause this slowing? (Select all that apply.) 1. Infiltration at VAD site 2. Patient lying on tubing 3. Roller clamp wide open 4. Tubing kinked in bedrails 5. Circulatory overload

1. Infiltration at VAD site 2. Patient lying on tubing 4. Tubing kinked in bedrails Factors that could slow an IV infusion even if the infusion pump is set correctly include increased pressure at the outflow site (e.g., infiltration) and compression of the tubing lumen (e.g., patient lying on the tubing or tubing kinked in bedrails).

A 20-year-old patient diagnosed with an eating disorder has a nursing diagnosis of Situational Low Self-Esteem. Which of the following nursing interventions are appropriate to address self-esteem? (Select all that apply.) 1. Offer independent decision-making opportunities. 2. Review previously successful coping strategies. 3. Provide a quiet environment with minimal stimuli. 4. Support a dependent role throughout treatment. 5. Increase calorie intake to promote weight stabilization.

1. Offer independent decision-making opportunities. 2. Review previously successful coping strategies Offering opportunities for decision making promotes a sense of control, which is essential for promoting independence and enhancing self-esteem. Reviewing successful coping strategies is also a priority intervention to signal previous mastery and promote effective coping in an individual with self-esteem issues.The amount of stimuli is unrelated to self-esteem. Promoting independence is an important part of treatment. Although weight stabilization may be needed, it is likely to have a negative effect on self-esteem early in treatment.

During a nursing assessment a patient displayed several behaviors. Which behavior suggests the patient may have a health literacy problem? 1. Patient has difficulty completing a registration form at a medical office 2. Patient asks for written information about a health topic 3. Patient speaks Spanish as primary language 4. Patient states unfamiliarity with a newly ordered medicine

1. Patient has difficulty completing a registration form at a medical office Behaviors that might reflect a health literacy deficit include having difficulty completing registration forms or health histories, failing to make follow-up appointments, and asking few questions during a nursing history or physical examination.

The nurse has just witnessed her patient go into cardiac arrest. The family is in the patient's room at the time the cardiac arrest occurs. What priority interventions should the nurse perform at this time? (Select all that apply.) 1. Perform chest compressions. 2. Ask someone to bring the automatic external defibrillator (AED) to the room for immediate defibrillation. 3. Apply oxygen via nasal cannula. 4. Place the patient supine. 5. Educate the family about the need for CPR

1. Perform chest compressions. 2. Ask someone to bring the automatic external defibrillator (AED) to the room for immediate defibrillation. 4. Place the patient supine The nurse needs to initiate CPR and chest compressions. The patient needs to be in a supine position for chest compressions to be effective. Applying a nasal cannula is ineffective as rescue breathing must be implemented. The family does need to be educated, but this is not the priority for the nurse at this time. The nurse should delegate escorting the family from the room to an assistive personnel. Educating the family about CPR can be delegated to another nurse who is not actively engaged in the resuscitation

A nurse is conducting a home visit with a new mom and her three children. While in the home the nurse weighs each family member and reviews their 3-day food diary. She checks the mom's blood pressure and encourages the mom to take the children for a 30-minute walk every day. The nurse is addressing which level of need, according to Maslow's Hierarchy of Needs? 1. Physiological 2. Safety and security 3. Love and belonging 4. Self-actualization

1. Physiological The nurse's actions address the basic physiological needs of nutrition, physical activity, and oxygen. According to Maslow's Hierarchy of Needs, basic needs must be met before meeting higher-level needs

Health care agencies must provide which of the following based on federal civil rights laws? (Select all that apply.) 1. Provide language assistance services at all points of contact free of charge. 2. Provide auxiliary aids and services, such as interpreters, note takers, and computer-aided transcription services. 3. Use patients' family members to interpret difficult topics. 4. Ensure that interpreters are competent in medical terminology. 5. Provide language assistance to all patients who speak limited English or are deaf.

1. Provide language assistance services at all points of contact free of charge. 2. Provide auxiliary aids and services, such as interpreters, note takers, and computer-aided transcription services. 4. Ensure that interpreters are competent in medical terminology. 5. Provide language assistance to all patients who speak limited English or are deaf.The CLAS standards include standards for communication and language assistance, including providing language assistance free of charge, auxiliary aids and services, interpreters competent in medical terminology, and language assistance for patients with limited English or who are deaf.

A patient who is depressed is crying and verbalizes feelings of low self-esteem and self-worth, such as "I'm such a failure. I can't do anything right." What is the nurse's best response? 1. Remain with the patient until the patient validates feeling more stable. 2. Tell the patient that this is not true and that every person has a purpose in life. 3. Review recent behaviors or accomplishments that demonstrate skill ability. 4. Reassure the patient that you know how he or she is feeling and that things will get better.

1. Remain with the patient until the patient validates feeling more stable. Demonstrating acceptance of the patient by supportively sitting with the patient builds a therapeutic nurse-patient relationship. The nurse's presence signals value and allows the patient to explore issues of self-concept and self-esteem. In contrast, giving false hope is neither therapeutic nor conveys acceptance, and focusing on skill ability signals conditional approval.

What are the 5 rights of delegation?

1. Right task 2. Right circumstance 3. Right person 4. Right direction/communication 5. Right supervision/evaluation

A nurse asks an AP to help the patient in Room 418 walk to the bathroom right now. The nurse tells the AP that the patient needs the assistance of one person and the use of a walker. The nurse also tells the AP that the patient's oxygen can be removed while he goes to the bathroom but to make sure that when it is put back on, the flowmeter is still at 2 L. The nurse also instructs the AP to make sure the side rails are up and the bed alarm is reset after the patient gets back in bed. Which of the following components of the "Five Rights of Delegation" were used by the nurse? (Select all that apply.) 1. Right task 2. Right circumstance 3. Right person 4. Right directions and communication 5. Right supervision and evaluation

1. Right task 2. Right circumstance 3. Right person 4. Right directions and communication The nurse provided 4 of the 5 components but did not provide the right supervision and evaluation. The nurse delegated the task of assisting a patient to the bathroom to the AP, which is in the scope of an AP's duties and responsibilities and matched to the AP skill level. The nurse did provide clear directions by describing the task and the time period to complete the task. The nurse did not use "please" and "thank you" in the request. The nurse did not ask whether there were any questions, which would provide the AP an opportunity to get clarification if needed. The nurse did not ask the AP to follow up on how the patient did or whether there were any problems. The nurse did not provide approp

A nurse is teaching an older-adult patient about ways to detect a melanoma. Which of the following are age-appropriate teaching techniques for this patient? (Select all that apply.) 1. Speak in a low tone. 2. Begin and end the session with the most important information regarding melanoma. 3. Provide a pamphlet about melanoma with large font in blues and greens. 4. Use basic one- or two-syllable medical terms. 5. Provide specific information in frequent, small amounts. 6. Speak quickly so that you do not take up much of the patient's time

1. Speak in a low tone. 2. Begin and end the session with the most important information regarding melanoma. 5. Provide specific information in frequent, small amounts. Lower tones are easier for patients with hearing deficits to hear. Reinforce important information at the beginning and end of each teaching session to enhance understanding. Providing information in small amounts helps the older adult understand information better. A pamphlet should be written in a color that contrasts with the background (e.g., black 14-point print on matte white paper). Blues and greens are hard to distinguish. Take your time with an older adult. Speaking quickly can easily lead to misunderstanding. An older adult does not require short-syllable terms unless there is evidence of lower health literacy

The nurse in a new community-based clinic is requested to complete a community assessment. Order the steps for completing this assessment. 1. Structure or locale 2. Social systems 3. Population

1. Structure or locale 3. Population 2. Social systems To begin a community assessment, the structure and geographic boundaries of the community are identified. Look at the structures in the community (e.g., schools, churches, types of residences). Next, obtain data about the population and the demographics of the community. Who are the residents of the community, what is the age range, and which types of ethnicity are represented? Last, review the social systems in the community

A new nurse graduate is in orientation on a surgical unit and is being mentored by an experienced nurse. Which action completed by the new nurse graduate requires intervention by the experienced nurse? (Select all that apply.) 1. The new nurse stops documenting about a dressing change to take a patient some water. 2. The new nurse gathered the medications for two different patients at the same time. 3. The new nurse asked an AP to help transfer a patient from the bed to a wheelchair before discharge. 4. The new nurse educates a patient about pain management when administering a pain medication to a patient. 5. The new nurse gathers all equipment necessary to start a new IV site before entering a patient's room.

1. The new nurse stops documenting about a dressing change to take a patient some water. 2. The new nurse gathered the medications for two different patients at the same time. Organizational skills help a nurse deliver care safely and effectively. In answer choice 1, the nurse interrupts documentation to attend to a basic patient need. In this situation, the nurse could have asked an AP to get the water or completed documentation and then gotten the water for the patient. Limiting interruptions is important in preventing errors. In answer choice 2, the nurse gathers medications for two patients at one time. Nurses should prepare medications for only one patient at a time to prevent medication errors.

The nurse manager from the surgical unit was awarded the nursing leadership award for practice of transformational leadership. Which of the following are characteristics or traits of transformational leadership displayed by the award winner? (Select all that apply.) 1. The nurse manager regularly rounds on staff to gather input on unit decisions. 2. The nurse manager sends thank-you notes to staff in recognition of a job well done. 3. The nurse manager sends memos to staff about decisions that the manager has made regarding unit policies. 4. The nurse manager has an "innovation idea box" to which staff are encouraged to submit ideas for unit improvements. 5. The nurse manager develops a philosophy of care for the staff

1. The nurse manager regularly rounds on staff to gather input on unit decisions. 2. The nurse manager sends thank-you notes to staff in recognition of a job well done. 4. The nurse manager has an "innovation idea box" to which staff are encouraged to submit ideas for unit improvements. Nurse managers who practice transformational leadership are focused on change and innovation. They motivate and empower their staffs with a focus on team development. The manager will spend time on the unit with the staff, sharing ideas and listening to staff input. The manager is enthusiastic about opportunities to enhance the team and shows appreciation and recognizes team members for good work. The manager holds the team accountable and provides support for the team members in the stressful health care environment. A transformational leader collaborates with the nursing staff to create a philosophy of care for their work unit.

The nurse is caring for a patient with an artificial airway. What are reasons to suction the patient? (Select all that apply.) 1. The patient has visible secretions in the airway. 2. There is a sawtooth pattern on the patient's EtCO2 monitor. 3. The patient has clear breath sounds. 4. It has been 3 hours since the patient was last suctioned. 5. The patient has excessive coughing.

1. The patient has visible secretions in the airway. 2. There is a sawtooth pattern on the patient's EtCO2 monitor. 5. The patient has excessive coughing. Refer to Skill 41.1. Clear breath sounds are normal and do not indicate the need for suctioning. Suctioning should be based upon assessment findings and not performed on a time-oriented basis.

When assessing a patient's adjustment to the role changes brought about by a medical condition such as a stroke, which of the following are critical assessment questions? (Select all that apply.) 1. What are your thoughts about returning to work? 2. What questions do you have about your medications? 3. How has your health affected your relationship with your partner? 4. What level of physical activity are you able to perform? 5. What concerns do you have about another stroke?

1. What are your thoughts about returning to work? 3. How has your health affected your relationship with your partner? The nurse must assess role performance as related to professional identity (work) and personal relationships (partner). The other questions are important but not related to self-concept.

A patient's cultural background affects the motivation for learning. Using the ACCESS model, match the nursing approach with the correct model component. ACCESS Model Component Nursing Approach __1. Assessment __2. Communication __3. Cultural __4. Establishment __5. Sensitivity __6. Safety A. Help patients feel culturally secure and able to maintain their cultural identity. B. Remain aware of verbal and nonverbal responses. C. Be aware of how patients from diverse backgrounds perceive their care needs. D. Become aware of your patient's culture and your own cultural biases. E. Learn about the patient's health beliefs and practices. F. Show respect by creating a caring rapport.

1E, 2B, 3D, 4F, 5C, 6A. 1. Assessment - E. Learn about the patient's health beliefs and practices. 2. Communication - B. Remain aware of verbal and nonverbal responses. 3. Cultural - D. Become aware of your patient's culture and your own cultural biases. 4. Establishment -F. Show respect by creating a caring rapport. 5. Sensitivity - C. Be aware of how patients from diverse backgrounds perceive their care needs. 6. Safety - A. Help patients feel culturally secure and able to maintain their cultural identity.

A nurse working on a medical patient care unit states, "I am having trouble sleeping, and I eat nonstop when I get home. All I can think of when I get to work is how I can't wait for my shift to be over. I wish I felt happy again." What are the best responses from the nurse manager? (Select all that apply.) 1. "I'm sure this is just a phase you are going through. Hang in there. You'll feel better soon." 2. "I know several nurses who feel this way every now and then. Tell me about the patients you have cared for recently. Did you find it difficult to care for them?" 3. "You can take diphenhydramine over the counter to help you sleep at night." 4. "Describe for me what you do with your time when you are not working." 5. "The hospital just started a group where nurses get together to talk about their feelings. Would you like for me to e-mail the schedule to you?"

2. "I know several nurses who feel this way every now and then. Tell me about the patients you have cared for recently. Did you find it difficult to care for them?" 4. "Describe for me what you do with your time when you are not working." 5. "The hospital just started a group where nurses get together to talk about their feelings. Would you like for me to e-mail the schedule to you?" This nurse is experiencing symptoms of compassion fatigue. The nurse manager needs to establish a therapeutic relationship with the nurse. Acknowledging personal thoughts and feelings and talking with other nurses to identify coping strategies can help this nurse work through the feelings associated with compassion fatigue. Engaging in healthy behaviors and establishing a good work-life balance may also help.

As part of a faith community nursing program in her church, a nurse is developing a health promotion program on breast selfexamination for the women's group. Which statement made by one of the participants is related to the individual's accurate perception of susceptibility to an illness? 1. "I have a hanging tag on my bathroom door to remind me to do my breast self-examination monthly." 2. "Since my mother had breast cancer, I know that I am at increased risk for developing breast cancer." 3. "Since I am only 25 years of age, the risk of breast cancer for me is very low." 4. "I participate every year in our local walk/run to raise money for breast cancer research."

2. "Since my mother had breast cancer, I know that I am at increased risk for developing breast cancer." On the basis of the health belief model, this statement indicates that the patient is concerned about developing breast cancer and feels that there is a risk or susceptibility based on recognition of a familial link for the disease. Once this link is recognized, the patient may perceive the personal risk.

The nurse is performing discharge teaching for a patient with chronic obstructive pulmonary disease (COPD). What statement, made by the patient, indicates the need for further teaching? 1. "Pursed-lip breathing is like exercise for my lungs and will help me strengthen my breathing muscles." 2. "When I am sick, I should limit the amount of fluids I drink so that I don't produce excess mucus." 3. "I will ensure that I receive an influenza vaccine every year, preferably in the fall." 4. "I will look for a smoking-cessation support group in my neighborhood."

2. "When I am sick, I should limit the amount of fluids I drink so that I don't produce excess mucus." Patients need to make sure that they are adequately hydrated to liquefy secretions, making it easier to expectorate. Fluids should not be limited or else the mucus will become too thick. All the other answers indicate an understanding of the discharge plan

The health care provider's order is 500 mL 0.9% NaCl intravenously over 4 hours. Which rate does the nurse program into the infusion pump? 1. 100 mL/hr 2. 125 mL/hr 3. 167 mL/hr 4. 200 mL/hr

2. 125 mL/hr To infuse 500 mL in 4 hours, set the rate at 125 mL/h (500 ÷ 4 5 125).

A nurse received change-of-shift reports on these four patients and starts rounding. Which patient does the nurse need to focus on as a priority? 1. A patient who had abdominal surgery 2 days ago and is requesting pain medication 2. A patient admitted yesterday with atrial fibrillation who now has a decreased level of consciousness 3. A patient with a wound drain who needs teaching before discharge in the early afternoon 4. A patient going to surgery for a mastectomy in 3 hours who has a question about the surgery

2. A patient admitted yesterday with atrial fibrillation who now has a decreased level of consciousness This patient is of high priority. The patient is experiencing the physiological problem of decreased level of consciousness that is an immediate threat to the patient's survival and safety. The nurse must intervene promptly and notify the health care provider of the life-threatening problem

Which of the following scenarios demonstrate that patient learning has taken place? (Select all that apply.) 1. A patient listens to a nurse's review of the warning signs of a stroke. 2. A patient describes how to set up a pill organizer for newly ordered medicines. 3. A patient attends a spinal cord injury support group. 4. A patient demonstrates how to take his blood pressure at home. 5. A patient reviews written information about resources for cancer survivors.

2. A patient describes how to set up a pill organizer for newly ordered medicines. 4. A patient demonstrates how to take his blood pressure at home. Steps 2 and 4 are examples of patients exhibiting behaviors that demonstrate learning. The other three steps are examples of patient involvement in instruction.

Which nursing interventions should a nurse implement when removing an indwelling urinary catheter in an adult patient? (Select all that apply.) 1. Attach a 3-mL syringe to the inflation port. 2. Allow the balloon to drain into the syringe by gravity. 3. Initiate a voiding record/bladder diary. 4. Pull the catheter quickly. 5. Clamp the catheter before removal.

2. Allow the balloon to drain into the syringe by gravity. 3. Initiate a voiding record/bladder diary. By allowing the balloon to drain by gravity, it is possible to avoid the development of creases or ridges in the balloon and thus minimize trauma to the urethra during withdrawal. All patients who have a catheter removed should have their voiding monitored. The best way to do this is with a voiding record or bladder diary. The size of the syringe used to deflate the balloon is dictated by the size of the balloon. In the adult patient balloon sizes are either 10 mL or 30 mL. Catheters should be pulled out slowly and smoothly. There is no evidence to support clamping catheters before removal.

A patient with a three-way indwelling urinary catheter and CBI complains of lower abdominal pain and distention after surgery. What should be the nurse's initial intervention(s)? (Select all that apply.) 1. Increase the rate of the CBI. 2. Assess the patency of the drainage system. 3. Measure urine output. 4. Assess vital signs. 5. Administer ordered pain medication.

2. Assess the patency of the drainage system. 3. Measure urine output. An appropriate first action would be to assess the patency of the drainage system. Urine output in the drainage bag should be more than the volume of the irrigant solution infused. If the system is not draining urine and irrigant, the irrigant should be stopped immediately; the catheter may be occluded, and the bladder distended. Pain medication should not be administered until after assessment is completed.

After abdominal surgery, the patient is on the surgical unit with an indwelling urinary catheter placed. What aspects of care for this patient can be delegated to the assistive personnel (AP)? (Select all that apply.) 1. Assessing the patient for any postoperative issues with the indwelling catheter 2. Assisting the nurse with patient positioning and maintaining privacy during catheter care 3. Teaching the patient signs and symptoms of a UTI 4. Reporting to the nurse any patient discomfort or fever 5. Reporting any abnormal color, odor, or amount of urine in the drainage bag

2. Assisting the nurse with patient positioning and maintaining privacy during catheter care 4. Reporting to the nurse any patient discomfort or fever 5. Reporting any abnormal color, odor, or amount of urine in the drainage bag The AP may assist the nurse with patient positioning, focus lighting for the procedure, maintain privacy, empty urine from collection bag, and help with perineal care. The AP may report postprocedure patient discomfort or fever to the nurse or any abnormal color, odor, or amount of urine in drainage bag and if the catheter is leaking or causes pain. The AP may not assess, teach, or evaluate aspects of patient care

Which assessment findings indicate that the patient is experiencing an acute disturbance in oxygenation and requires immediate intervention? (Select all that apply.) 1. SpO2 value of 95% 2. Chest retractions 3. Respiratory rate of 28 breaths per minute 4. Nasal flaring 5. Clubbing of fingers

2. Chest retractions 3. Respiratory rate of 28 breaths per minute 4. Nasal flaring found in Table 41.2. SpO2 of 95% is normal and requires no intervention. Clubbed fingers are an assessment finding associated with chronic hypoxia; this does not require immediate intervention.

A hospice nurse is caring for a family that is providing end-of-life care for their grandmother, who has terminal breast cancer. The nurse focuses on symptom management for the grandmother and on helping the family with developing coping skills. This approach is an example of which of the following? 1. Family as context 2. Family as patient 3. Family as a system 4. Family as structure

2. Family as patient When the family as patient is the approach, the family's needs, processes, and relationships (e.g., parenting or family caregiving) are the primary focuses of nursing care.

At 1300 the registered nurse (RN) says to the assistive personnel (AP), "You did a good job transferring Mr. Harvey into his chair this morning at 0830. I saw that you recorded how long he stayed in his chair. I saw that Mr. Harvey did not have any shoes or nonslip slippers on, though. For safety, the next time you transfer a patient into a chair, you need to make sure that the patient wears slippers or shoes. Please get Mr. Harvey up in his chair again by 1500." Which characteristics of positive feedback did the RN use when talking to the AP? (Select all that apply.) 1. Feedback is given immediately. 2. Feedback focuses on one issue. 3. Feedback offers concrete details. 4. Feedback identifies ways to improve. 5. Feedback focuses on changeable things. 6. Feedback is specific about what is done incorrectly only

2. Feedback focuses on one issue. 3. Feedback offers concrete details. 4. Feedback identifies ways to improve. 5. Feedback focuses on changeable things. These are characteristics of good feedback. The nurse gives feedback on the process of the AP monitoring and ambulating a patient. The other options are inappropriate because the RN did not provide feedback immediately (the AP performed the task in the morning, but the feedback was not given until the afternoon), and the nurse should give both positive feedback as well as feedback to improve the incorrectly done tasks

Which nursing intervention decreases the risk for CAUTI? 1. Cleansing the urinary meatus 3 to 4 times daily with antiseptic solution 2. Hanging the urinary drainage bag below the level of the bladder 3. Emptying the urinary drainage bag daily 4. Irrigating the urinary catheter with sterile water

2. Hanging the urinary drainage bag below the level of the bladder Evidence-based interventions shown to decrease the risk for CAUTI include ensuring that there is a free flow of urine from the catheter to the drainage bag. This will occur when the drainage bag is hung below the level of the bladder.

During a visit to a family clinic, a nurse teaches a mother about immunizations, the use of car seats, and home safety for an infant and toddler. Which type of nursing interventions are these? 1. Restorative 2. Health promotion 3. Acute care 4. Growth and development

2. Health promotion - activities focus on interventions designed to maintain the physical, social, emotional, and spiritual health of the family unit, including information about specific health behaviors and family coping techniques.

8. Which of the following are symptoms of secondary traumatic stress and burnout that commonly affect nurses? (Select all that apply.) 1. Regular participation in a book club 2. Lack of interest in exercise 3. Difficulty falling asleep 4. Lack of desire to go to work 5. Anxiety while working

2. Lack of interest in exercise 3. Difficulty falling asleep 4. Lack of desire to go to work 5. Anxiety while working Nurses are particularly susceptible to the development of secondary traumatic stress and burnout—the components of compassion fatigue. Symptoms include decline in health, emotional exhaustion, irritability, restlessness, impaired ability to focus and engage with patients, feelings of hopelessness, inability to take pleasure from activities, and anxiety

The nurse is inserting a urinary catheter for a female patient, and after the catheter has been inserted 3 inches, no urine is returned. What should the nurse do next? 1. Remove the catheter and start all over with a new kit and catheter. 2. Leave the catheter there and start over with a new catheter. 3. Pull the catheter back and reinsert at a different angle. 4. Ask the patient to bear down and insert the catheter farther.

2. Leave the catheter there and start over with a new catheter. The catheter may be in the vagina; leave the catheter in the vagina as a landmark indicating where not to insert and insert another sterile catheter. Pulling the catheter back and reinserting is poor technique, increasing the risk for catheter-associated urinary tract infection (CAUTI).

. JoAnn, a nurse, is calling a patient's health care provider about a problem that her patient, Ms. Ducote, is having during a blood transfusion. The health care agency uses the SBAR system in reporting patient problems. Put the statements in the correct order according to the SBAR system. 1. I think she might need a diuretic ordered. Is it possible for you to come see her soon? Is there anything else you would like for me to do right now? 2. This is JoAnn. I am caring for Ms. Ducote. She is having labored breathing and her heart rate is higher now than it was an hour ago. She is receiving the second unit of her blood transfusion. She says she is having trouble catching her breath. 3. Ms. Ducote had surgery earlier yesterday to remove a tumor in the colon. Her hemoglobin was 9.6 grams/dL, and her hematocrit was 33.6% this morning. Her first ordered unit of packed red blood cells infused over 90 minutes, and she is 30 minutes into receiving her second unit of blood now. She states she takes furosemide 20 mg every morning at home. She does not currently have an order for furosemide. 4. The patient denies pain, and her vital signs are as follows: B/P 150/98 mm Hg; pulse 118; respiratory rate 28; temperature 98.8° F (37.1° C). I think she is showing signs of fluid volume overload.

2. This is JoAnn. I am caring for Ms. Ducote. She is having labored breathing and her heart rate is higher now than it was an hour ago. She is receiving the second unit of her blood transfusion. She says she is having trouble catching her breath. 3. Ms. Ducote had surgery earlier yesterday to remove a tumor in the colon. Her hemoglobin was 9.6 grams/dL, and her hematocrit was 33.6% this morning. Her first ordered unit of packed red blood cells infused over 90 minutes, and she is 30 minutes into receiving her second unit of blood now. She states she takes furosemide 20 mg every morning at home. She does not currently have an order for furosemide. 4. The patient denies pain, and her vital signs are as follows: B/P 150/98 mm Hg; pulse 118; respiratory rate 28; temperature 98.8° F (37.1° C). I think she is showing signs of fluid volume overload. 1. I think she might need a diuretic ordered. Is it possible for you to come see her soon? Is there anything else you would like for me to do right now? SBAR provides a consistent way to communicate patient problems. In this example, the Situation (S) is that the patient is experiencing labored breathing. Next the nurse provides the Background (B) about the patient's surgery, current orders, and how much blood she has received so far. Then the nurse provides Assessment (A) data about the patient's current status. Finally, the nurse provides a Recommendation (R) to administer a diuretic and requests the health care provider to come see the patient soon.

A nursing student is giving a presentation to a group of other nursing students about the needs of patients with mental illnesses in the community. Which statement by the student indicates that the nursing professor needs to provide further teaching? 1. "Many patients with mental illness do not have a permanent home." 2. "Unemployment is a common problem experienced by people with a mental illness." 3. "The majority of patients with mental illnesses live in longterm care settings." 4. "Patients with mental illnesses are often at a higher risk for abuse and assault."

3. "The majority of patients with mental illnesses live in longterm care settings." Many patients with severe mental illnesses are homeless, unemployed, and at a greater risk for abuse and assault. Patients with mental illnesses no longer routinely live in long-term psychiatric institutions but instead receive resources in the community.

Based on the Transtheoretical Model of Change, what is the most appropriate response to a patient who states: "Me, stop smoking? I've been smoking since I was 16!" 1. "That's fine. Some people who smoke live a long life." 2. "OK. I want you to decrease the number of cigarettes you smoke by one each day, and I'll see you in 1 month." 3. "What do you think is the greatest reason why stopping smoking would be challenging for you?" 4. "I'd like you to attend a smoking-cessation class this week and use nicotine replacement patches as directed."

3. "What do you think is the greatest reason why stopping smoking would be challenging for you?" The patient's response indicates that he is in the precontemplation stage and not intending to make a change in behavior in the next 6 months. In this stage the patient is not interested in information about the behavior and may be defensive when confronted with the information. Asking an open-ended question about smoking may stimulate the patient to identify a reason to begin a behavioral change. Nurses are challenged to motivate and facilitate health behavior change in working with individuals.

Which example demonstrates a nurse performing the skill of evaluation? 1. The nurse explains the side effects of the new blood pressure medication ordered for the patient. 2. The nurse asks a patient to rate pain on a scale of 0 to 10 before administering a pain medication. 3. After completing a teaching session, the nurse observes a patient drawing up and administering an insulin injection. 4. The nurse changes a patient's leg ulcer dressing using aseptic technique.

3. After completing a teaching session, the nurse observes a patient drawing up and administering an insulin injection. Evaluation is one of the most important aspects of clinical care coordination, involving the determination of patient outcomes. Observing a patient do a return demonstration of teaching is evaluation to ensure that patient has understood teaching. Option 2 is not evaluation because it occurs before administering a pain medication. The other options are interventions

A 26-year-old patient visits a medical clinic and asks a nurse to provide instruction on how to perform a breast self-examination. "My mom had cancer so I want to learn how." Which domains are required to learn this skill? (Select all that apply.) 1. Affective domain 2. Sensory domain 3. Cognitive domain 4. Attentional domain 5. Psychomotor domain

3. Cognitive domain 5. Psychomotor domain For a patient to perform a breast self-examination, it will be necessary to understand the purpose of the examination and why it is performed the way it is. Cognitive learning in this scenario involves a patient acquiring information to further develop understanding and thinking processes in order to make a decision based on a self-examination finding. Psychomotor learning in this case involves actual use of the hands to palpate in symmetrical areas of the breast correctly. Affective domain does not apply here unless the nurse decides the patient's values prevent self-examination ad

A 30-year-old patient diagnosed with major depressive disorder has a nursing diagnosis of Situational Low Self-Esteem related to negative view of self. Which of the following are appropriate interventions by the nurse? (Select all that apply.) 1. Encourage reconnecting with high school friends. 2. Role-play to increase assertiveness skills. 3. Focus on identifying strengths and accomplishments. 4. Provide time for journaling to explore underlying thoughts and feelings. 5. Explore new job opportunities

3. Focus on identifying strengths and accomplishments. 4. Provide time for journaling to explore underlying thoughts and feelings. Focusing on strengths and accomplishments to minimize the emphasis on failures helps the patient alter distorted and negative thinking.Journaling can allow a patient to explore thoughts and feelings that can promote insight and eventual behavioral change. The other interventions represent the nurse imposing ideas on what needs to occur for the patient to be healthier; allowing the patient to direct the change process is important.

Which assessment does the nurse use as a clinical marker of vascular volume in a patient at high risk of ECV deficit? 1. Dryness of mucous membranes 2. Skin turgor 3. Fullness of neck veins when supine 4. Fullness of neck veins when upright

3. Fullness of neck veins when supine ECV deficit involves decreased vascular and interstitial volume. One way to assess vascular volume is to examine the fullness of neck veins when an individual is supine. With ECV, neck veins are full when the individual is supine. With ECV deficit, they are flat.

A nurse performs the following four steps in delegating a task to an AP. Place the steps in the correct order of appropriate delegation. 1. Do you have any questions about walking with Mr. Malone? 2. Before you take him for his walk to the end of the hallway and back, please take and record his pulse rate. 3. In the next 30 minutes please assist Mr. Malone in Room 418 with his afternoon walk. 4. I will make sure that I check with you in about 40 minutes to see how the patient did.

3. In the next 30 minutes please assist Mr. Malone in Room 418 with his afternoon walk. 2. Before you take him for his walk to the end of the hallway and back, please take and record his pulse rate. 4. I will make sure that I check with you in about 40 minutes to see how the patient did. 1. Do you have any questions about walking with Mr. Malone? This is the sequence of effective delegation. The nurse delegated the task of walking a patient to the AP, which is in the scope of the AP's duties and responsibilities and matched to the AP's skill level. The nurse provided clear directions by describing the task (the walk, taking and recording the pulse), the desired outcome (walk to the end of the hallway and back), and the time period (within the next 30 minutes). The nurse explained the process of follow-up with the AP to check how the patient did. The nurse asked whether the AP had any questions to provide the AP the opportunity to ask questions for clarification.

A nurse who works in an outpatient chemotherapy infusion center is assigned to the care of a 56-year-old male patient who is receiving chemotherapy for colon cancer. This is the patient's first clinic visit. The nurse reviews the patient's medical record and sees a note about the patient receiving instruction on how chemotherapy treats cancer; the note does not summarize the patient's response. The nurse's assessment reveals that the patient is motivated to learn more about his chemotherapy and is alert and currently feeling well. The nurse has prepared the first infusion, regulated it, and now takes time to begin instruction. Which teaching approaches are best suited for this situation? 1. Use an entrusting approach in explaining how to monitor for side effects of chemotherapy. 2. Bring the patient together with two other patients in the clinic and conduct a group discussion. 3. Provide verbal one-on-one instruction, with the patient participating in selection of content. 4. Present the patient with a problem situation involving a serious side effect and have the patient decide what to do

3. Provide verbal one-on-one instruction, with the patient participating in selection of content. Because this is the first instructional session with this patient, the nurse should choose one-on-one instruction and participation. This will set the groundwork for future teaching sessions. It is too early to use an entrusting approach or to present a problem situation. A group approach is also too early but could be valuable in future visits.

Which instruction should the nurse give the assistive personnel (AP) concerning a patient who has had an indwelling urinary catheter removed that day? 1. Limit oral fluid intake to avoid possible UI. 2. Expect patient complaints of suprapubic fullness and discomfort. 3. Report the time and amount of first voiding. 4. Instruct patient to stay in bed and use a urinal or bedpan.

3. Report the time and amount of first voiding. To adequately assess bladder function after a catheter is removed, voiding frequency and amount should be monitored. Unless contraindicated, fluids should be encouraged. To promote normal micturition, patients should be placed in as normal a posture for voiding as possible. Suprapubic tenderness and pain are possible indicators of urinary retention and/or a urinary tract infection

A family includes a mother, a stepfather, two teenage biological daughters of the mother, and a biological daughter of the father. The father's daughter just moved home following the loss of her job in another city. The family is converting a study into a bedroom and is in the process of distributing household chores. Nursing assessment reveals that all members of the family think that their family can adjust to lifestyle changes. This is an example of family: 1. Diversity 2. Durability 3. Resiliency 4. Configuration

3. Resiliency is the ability of the family to cope with the unexpected. This family used resources to provide some short-term solutions for the return home of an adult child.

A patient is admitted to the hospital with severe dyspnea and wheezing. ABG levels on admission are pH 7.26; PaO2, 68 mm Hg; PaCO2, 55 mm Hg; and HCO3 2, 24. How does the nurse interpret these laboratory values? 1. Metabolic acidosis 2. Metabolic alkalosis 3. Respiratory acidosis 4. Respiratory alkalosis

3. Respiratory acidosis The pH is abnormally low, which indicates acidosis. The PaCO2 is high, which indicates respiratory acidosis. The HCO3 2 is in the normal range, which indicates an acute respiratory acidosis that has not had time for renal compensation. The low PaO2 and the severe dyspnea and wheezing are consistent with this interpretation.

A patient recovering from open heart surgery is taught how to cough and deep breathe using a pillow to support or splint the chest incision. Following the teaching session, which of the following is the best way for the nurse to evaluate whether learning has taken place? 1. Verbalization of steps to use in splinting 2. Selecting from a series of flash cards the images showing the correct technique 3. Return demonstration 4. REALM test

3. Return demonstration permits a patient to perform a skill as the nurse observes. It provides excellent feedback and reinforcement

A nurse is preparing to teach a patient who has sleep apnea how to use a continuous positive airway pressure (CPAP) machine at night. Which action is most appropriate for the nurse to perform first? 1. Allow patient to manipulate machine and look at parts. 2. Provide a teach-back session. 3. Set mutual goals for the education session. 4. Discuss the purpose of the machine and how it works.

3. Set mutual goals for the education session. Planning should occur before any form of implementation or evaluation. The nurse should build from simple to more complex information. Learning about the purpose of the machine and how it works is basic information needed for the patient to understand and be motivated to use it. Allowing the patient to manipulate the machine will precede instruction on its actual use. Teach-back will inform the nurse as to the patient's level of learning

A patient discharged a week ago following a stroke is currently participating in rehabilitation sessions provided by nurses, physical therapists, and registered dietitians in an outpatient setting. In what level of prevention is the patient participating? 1. Primary prevention 2. Secondary prevention 3. Tertiary prevention 4. Transtheoretical prevention

3. Tertiary prevention involves minimizing the effects of long-term disease or disability by interventions directed at preventing complications and deterioration. Tertiary prevention activities are directed at rehabilitation rather than diagnosis and treatment. Care at this level aims to help patients achieve as high a level of functioning as possible, despite the limitations caused by illness or impairment. This level of care is called preventive care because it involves preventing further disability or reduced functioning

Using the Transtheoretical Model of Change, order the steps that a patient goes through to make a lifestyle change related to physical activity. 1. The individual recognizes that he is out of shape when his daughter asks him to walk with her after school. 2. Eight months after beginning walking, the individual participates with his wife in a local 5K race. 3. The individual becomes angry when the physician tells him that he needs to increase his activity to lose 30 pounds. 4. The individual walks 2 to 3 miles, 5 nights a week, with his wife. 5. The individual visits the local running store to purchase walking shoes and obtain advice on a walking plan.

3. The individual becomes angry when the physician tells him that he needs to increase his activity to lose 30 pounds. 1. The individual recognizes that he is out of shape when his daughter asks him to walk with her after school. 5. The individual visits the local running store to purchase walking shoes and obtain advice on a walking plan. 4. The individual walks 2 to 3 miles, 5 nights a week, with his wife. 2. Eight months after beginning walking, the individual participates with his wife in a local 5K race. t This sequence follows the order of the steps of the transtheoretical model of change: precontemplation, contemplation, preparation, action, and maintenance.

A nurse working in a large occupational health clinic knows that many of the workers at her company are marginalized and at risk for poor health outcomes. Which of the following individuals are most likely to be marginalized? 1. Wives of the employees 2. The head supervisors of the company 3. Workers who have a high school education 4. Workers employed for less than a year at the company

3. Workers who have a high school education Marginalized groups are more likely to have poor health outcomes and die earlier because of a complex interaction among their individual behaviors, environment of the communities in which they live, the policies and practices of health care and governmental systems, and the clinical care they receive. Examples of marginalized groups include people who are gay, lesbian, bisexual, or transgender; people of color; people who are physically and/or mentally challenged; and people who are not college educated

A 50-year-old woman is recovering from a bilateral mastectomy. She refuses to eat, discourages visitors, and pays little attention to her appearance. One morning the nurse enters the room to see the patient with her hair combed and makeup applied. Which of the following is the best response from the nurse? 1. "What's the special occasion?" 2. "You must be feeling better today." 3. "This is the first time I've seen you look this good." 4. "I see that you've combed your hair and put on makeup."

4. "I see that you've combed your hair and put on makeup." When the nurse uses a matter-of-fact approach and acknowledges a change in the patient's behavior or appearance, it allows the patient to establish its meaning. Telling the patient she has never looked this good conveys criticism; making assumptions about it being a special occasion or about an obvious improvement in mood superimposes the nurse's opinion and limits the assessment.

A 7-year-old child was recently diagnosed with asthma. A nurse is providing education to the child and her parents about the treatment and management of asthma and changes they need to make in their home environment to promote her health. Which statement made by the parents requires follow-up by the nurse? 1. "We've made an appointment to talk with the school nurse about the change in our child's health." 2. "We forgot to give our daughter her medications before bedtime, so we made a list of her medications to help us remember." 3. "We worked out a schedule to check on her before and after school." 4. "We haven't been spending time with our parents because we're so busy taking care of our daughter."

4. "We haven't been spending time with our parents because we're so busy taking care of our daughter." Changes in family relationships indicate possible family caregiver stress.

A 55-year-old man has been in the hospital for over a week following surgical complications. The patient has had limited activity but is now ordered to begin a mobility program. The patient just returned from several diagnostic tests and tells the nurse he is feeling fatigued. The nurse prepares to instruct the patient on the mobility program protocol. Which of the following learning principles will likely be affected by this patient's condition? 1. Motivation to learn 2. Developmental stage 3. Stage of grief 4. Readiness to learn

4. A patient's readiness to learn is affected by the patient's attentional set. Physical discomfort, fatigue, anxiety, confusion, and environmental distractions influence the ability to concentrate and learn.

A nurse in an outpatient setting is performing an admission assessment on a 77-year-old woman who was suddenly widowed 3 weeks ago. During the health history the patient tells the nurse that at times she "feels lost and isolated" and says she "does not feel like participating in previous social or hobby activities." She says she "just doesn't feel like her usual self." The nurse needs to understand that which of the following most directly influences the patient's current self-concept? 1. Attitude and behaviors of relatives providing care 2. Caring behaviors of the nurse and health care team 3. Level of education, economic status, and living conditions 4. Adjustment to role change, loss of loved ones, and physical energy

4. Adjustment to role change, loss of loved ones, and physical energy Older adults experience significant challenges to selfconcept, including mental and physical changes associated with aging and changes in identity and role following retirement and/or loss of significant others. The adjustment to stressors is most important. The other influences are important but to a lesser degree.

Which skills can the nurse delegate to assistive personnel (AP)? (Select all that apply.) 1. Initiate oxygen therapy via nasal cannula. 2. Perform nasotracheal suctioning of a patient. 3. Educate the patient about the use of an incentive spirometer. 4. Assist with care of an established tracheostomy tube. 5. Reposition a patient with a chest tube

4. Assist with care of an established tracheostomy tube. 5. Reposition a patient with a chest tube Assistive personnel (AP) are not allowed to initiate oxygen therapy, provide education, or perform NT suctioning on a patient. They are allowed to assist the nurse in performing tracheostomy tube care and with repositioning patients.

Place the following steps for discontinuing IV access in the correct order: 1. Perform hand hygiene and apply gloves. 2. Explain procedure to patient. 3. Remove IV site dressing and tape. 4. Use two identifiers to ensure correct patient. 5. Stop the infusion and clamp the tubing. 6. Carefully check the health care provider's order. 7. Clean the site, withdraw the catheter, and apply pressure

6. Carefully check the health care provider's order. 4. Use two identifiers to ensure correct patient. 2. Explain procedure to patient. 1. Perform hand hygiene and apply gloves. 5. Stop the infusion and clamp the tubing. 3. Remove IV site dressing and tape. 7. Clean the site, withdraw the catheter, and apply pressure A health care provider's order is necessary before discontinuing IV access, unless there is a complication such as infiltration or phlebitis. Identifying the patient and explaining the procedure are performed before hand hygiene and glove application in order to maintain clean gloves. Removing the site dressing before stopping the infusion and then withdrawing the catheter keeps the VAD patent without forming a clot that could embolize during catheter withdrawal

1. Beth should advance the catheter ________ to _______ inches or until urine flows out of it.

7 to 9 Rationale: In an adult male the catheter should be advanced 7 to 9 inches (17-22.5 cm) or until urine flows out of it.

The nurse is preparing to perform nasotracheal suctioning on a patient. Arrange the steps in order. 1. Apply suction. 2. Assist patient to semi-Fowler's or high Fowler's position, if able. 3. Advance catheter through nares and into trachea. 4. Have patient take deep breaths. 5. Lubricate catheter with water-soluble lubricant. 6. Apply sterile gloves. 7. Perform hand hygiene. 8. Withdraw catheter

7. Perform hand hygiene 2. Assist patient to semi-Fowler's or high Fowler's position, if able. 6. Apply sterile gloves. 4. Have patient take deep breaths 5. Lubricate catheter with water-soluble lubricant. 3. Advance catheter through nares and into trachea. 1. Apply suction 8. Withdraw catheter Refer to Skill 41.1 for the steps to this procedure.

Mrs. McIntosh is a 72-year-old Caucasian woman whose husband of 50 years recently passed away. She had been his primary caregiver. Since her husband's death 3 months ago, she has lost 20 pounds as a result of her lack of interest in food. She tells her daughter that she feels "so heavy and tired all of the time." Mrs. McIntosh, once vivacious and very active, now confesses that she has trouble getting out of bed in the morning because she doesn't have much purpose in life since she no longer needs to care for Mr. McIntosh. Mrs. McIntosh's daughter brings Mrs. McIntosh to the behavioral health clinic for assistance. Walter is the nursing student assigned to her. He takes a patient history and asks her several questions about her mood and recent habits. 1. Mrs. McIntosh no longer has to care for Mr. McIntosh. Because of this, she is most likely experiencing which of the following? A. Role performance stressor B. Body image disturbance C. Identity attainment D. Self-esteem freedom

A. Role performance stressor

3. Mrs. McIntosh is diagnosed with situational altered self-concept related to loss of husband. Walter develops the following goal for Mrs. McIntosh: "Patient's self-concept will improve in 2 weeks." Which of the following are examples of expected outcomes directed to this goal? (Select all that apply.) A. The patient will attend a bereavement support group twice a week. B. The patient will become less depressed. C. The patient will get out of bed each morning at 8 AM. D. The patient will bathe and dress each morning after rising. E. The patient will become more balanced.

A. The patient will attend a bereavement support group twice a week. C. The patient will get out of bed each morning at 8 AM. D. The patient will bathe and dress each morning after rising.

A patient comes from a close-knit family. The patient's family functions as context. You will need to evaluate: A. attainment of patient needs. B. family attainment of developmental traits C. individual family members caring about one another D. family satisfaction with its new level of functioning.

A. attainment of patient needs.

A nurse is caring for a client who has hypokalemia as an adverse effect of furosemide. Use the ATI Active Learning Template: System Disorder to complete this item to include the following sections. ALTERNATION IN HEALTH (DIAGNOSIS) EXPECTED FINDINGS: Identify at least five expected findings. NURSING CARE: Identify two nursing interventions for hypokalemia.

ALTERATION IN HEALTH (DIAGNOSIS): Hypokalemia is a blood potassium level less than 3.5mEQ/L that can result from the increased loss of potassium from the body due to the use of potassium-excreting diuretics (furosemide). EXPECTED FINDINGS: * Vital signs: weak irregular pulse, hypotension, respiratory distress *Neuromusculoskeletal: Ascending bilateral muscle weakness, muscle cramping, decreased muscle tone, hypoactive reflexes, paresthesia, mental confusion. *GI: Decreased motility, hypoactive bowel sounds, abdominal distention, constipation, nausea, vomiting, anorexia *Dysrhythmias: PVCs, bradycardia, blocks, ventricular tachycardia, flattening T waves, ST depression NURSING CARE * Monitor for cardiac dysrhythmia * Monitor for shallow or ineffective respirations * Teach and encourage consumption of potassium rich foods (bananas, avocados, cantaloupe). * Ensure the underlying cause of hypokalemia is corrected.

During a teaching session, the nurse tells a patient with a recent neck injury that damage to the nerves is comparable to a water hose that has been pinched off. During this teaching session, the nurse is using the process of: A. Analogy B. Discovery C. Role playing D. Demonstration

Analogy

Which number corresponds to the spot where you would assess for an air leak in the patient with a chest tube?

Answer #1 This component is the air leak monitor. Option 2 is the dry suction regulator, where a nurse can regulate or titrate the amount of suction applied to the chest tube. Option 3 is the suction monitor bellows that the nurse assesses to ensure that suction is working and being applied. If the bellows is all the way over to the right, then the suction is working. Option 4 is the collection chamber, where the nurse will monitor the amount or volume of drainage coming from the patient's pleural space.

1. Jeffrey tells Mrs. Lasky that it sounds as if she has ____________ into American culture since she has American friends and follows many of the American traditions.

Answer: assimilated Rationale: Assimilation occurs when a person is absorbed into a new culture.

1. Rochelle adds the social and family information to Mr. Sanders' existing care plan. She knows that family nursing practice involves three levels of approach. These levels of approach are the family as ___________, _________, and __________.

Answer: context, patient, system

1. Jay begins to plan Ms. Osborne's care and decides to delegate Ms. Osborne's assessment to the nursing assistant because he is behind on his patient rounds. It is acceptable for Jay to delegate the task of assessment according to the American Nurses Association (ANA) and the National Council of State Boards of Nursing (NCSBN). A. True B. False

B. False

1. Mr. Smith becomes agitated as Margaret tries to retrain him in his use of eating utensils. Margaret knows that, as his anxiety increases, his ability to pay attention also increases. A. True B. False

B. False

A nurse is caring for a client who is having difficulty breathing. The client is lying in bed and is already receiving oxygen therapy via nasal cannula. Which of the following interventions is the nurse's priority? A) Increase the oxygen flow B) Assist the client to Fowler's position C) Promote removal of pulmonary secretions D) Obtain a specimen for arterial blood gases.

B) Assist the client to Fowler's position

A nurse is collecting data for a client who has hypercalcemia as a result of long-term use of glucocorticoids. Which of the following findings should the nurse expect? (Select all that apply) A) Hyperreflexia B) Confusion C) Positive Chvostek's sign D) Bone pain E) Nausea and vomiting

B) Confusion: Expect the client who has hypercalcemia to have confusion and a possible decreased level of consciousness. D) Bone pain: Expect the client who has hypercalcemia to have bone pain. E) Nausea and vomiting: Expect the client who has hypercalcemia to have nausea, vomiting along with anorexia.

A home health nurse is discussing the dangers of food poisoning with a client. Which of the following information should the nurse include? (select all that apply.) A) Most food poisoning is caused by a virus. B) Immunocompromised individuals are at increased risk for complications for food poisoning. C) Clients who are at high risk should eat or drink only pasteurized dairy products. D) Healthy individuals usually recover from the illness in a few weeks E) Handling raw and fresh food separately can prevent food poisoning.

B) Immunocompromised individuals are at increased risk for complications for food poisoning. C) Clients who are at high risk should eat or drink only pasteurized dairy products. E) Handling raw and fresh food separately can prevent food poisoning.

A nurse is providing discharge instructions to a client who has a prescription for oxygen use at home. Which of the following information should the nurse include? (select all that apply.) A) Family members who smoke must be at least 10 ft from the client when oxygen is in use. B) Nail polish should not be used near a client who is receiving oxygen C) A "No Smoking" sign should be placed on the front door. D) Cotton bedding and clothing should be replaced with items made from wool. E) A fire extinguisher should be readily available in the home.

B) Nail polish should not be used near a client who is receiving oxygen C) A "No Smoking" sign should be placed on the front door. E) A fire extinguisher should be readily available in the home.

1. Ms. Jones and Mr. Sanders explain their relationship and living arrangement to Rochelle. Ms. Jones and Mr. Sanders are an example of which type of family form? A. Split B. Alternative C. Nuclear D. Blended E. Extended

B. Alternative

1. Immunizations are not an objective as defined by Healthy People 2030 because immunizations are largely for children and children are not included in the Healthy People 2030 initiative. A. True B. False

B. False

Franz is a first-semester nursing student who struggles with the concepts associated with electrolyte and acid-base balance. He knows that he needs to get a handle on these concepts because they are the foundation of understanding pathophysiology and as a nurse he will be responsible for understanding and interpreting laboratory reports so he may deliver the best patient care possible. Franz reviews the concepts of fluid, electrolyte, and acid-base balance as he studies for his midterm examination. 1. Franz learns that hyperkalemia may be the result of chronic diarrhea. A. True B. False

B. False

1. As Beth inserts the catheter into Mr. Kelter's penis, she feels resistance. She should use more force to guide the catheter through his urethra. A. True B. False

B. False Rationale: When resistance is felt while inserting a catheter, withdraw the catheter and do not force it. If there is resistance to the catheter insertion, have the patient take slow, deep breaths while slowly inserting the catheter. When urine appears, advance the catheter another 2.5 to 5 cm (1 to 2 inches).

1. Trevor knows that receiving an immunization is included in which hierarchy of need according to Maslow? A. Physiological B. Safety and security C. Love and belonging needs D. Self-esteem E. Self-actualization

B. Safety and security

Vulnerable populations of patients are those who are more likely to develop health problems as a result of: A. living at home B. abusive habits C. citizenship D. middle age

B. abusive habits

a newly graduated nurse is assigned to care for a team consisting of herself and a certified nursing assistant. When delegating skills, she needs to: A. assign only bed-making and feeding skills B. assess the knowledge of the CNA C. remind the staff member that she is working under the license of the RN D. allow the staff member to perform only skills that the RN is able to teach CNAs to perform

B. assess the knowledge of the CNA

You are in the process of admitting an ethically diverse patient. To plan culturally competent care, you will conduct a cultural assessment that includes: A. biocultural history B. ethnohistory C. negotiation D.ethnocentrism

B. ethnohistory

A travel nurse has taken an assignment at a health care facility where nurses assume responsibility for a caseload of patients over a period of time. This type of nursing exemplifies: A. team nursing. B. primary nursing. C. functional nursing. D. decentralized management.

B. primary nursing.

You are caring for an adolescent patient who underwent a gastric banding procedure 6 months previously. This adolescent tells you, "there is still a fat person inside of me". This type of statement illustrates a flow in the self-concept of: A. Identity B. Self esteem C. Body image D role performance

C body image

A nurse receives a laboratory report for a client indicating potassium level of 5.2 mEq/L. When notifying the provider, the nurse should expect which of the following actions? A) Starting an IV infusion of 0.9% sodium chloride B) Consulting with dietitian to increase intake in potassium C) Initiating continuous cardiac monitoring D) Preparing the client for a gastric lavage

C) Initiating continuous cardiac monitoring: a potassium level of 5.2 mEq/L indicates hyperkalemia. Anticipate the initiation of continuous cardiac monitoring due to the client's risk for dysrhythmias (ventricular fibrillation).

Mr. Ryan Kelter is a 33-year-old Caucasian who lives in an acute rehabilitation center. He was injured in a motorcycle accident that caused a spinal cord injury (SCI). As a result of the SCI, he has neurogenic bladder that prevents him from fully emptying his bladder. Because of this, he needs to be straight catheterized several times a day. Beth is the student nurse assigned to Mr. Kelter. She understands the importance of keeping him on his bladder schedule to prevent a urinary tract infection (UTI). 1. Beth enters Mr. Kelter's room after lunch to perform straight catheterization. List in order the steps Beth takes to perform straight catheterization on Mr. Kelter. A. Lubricate the catheter. B. Clean penis with dominant hand. C. Apply sterile gloves. D. Advance catheter into penis. E. Apply fenestrated drape. F. Hold penis with nondominant hand. G. Ask patient to bear down. H. Coil catheter in dominant hand.

C. Apply sterile gloves. E. Apply fenestrated drape. A. Lubricate the catheter. F. Hold penis with nondominant hand. B. Clean penis with dominant hand. H. Coil catheter in dominant hand. G. Ask patient to bear down. D. Advance catheter into penis. Rationale: The steps of straight catheterization are to position the patient, apply sterile gloves, apply the fenestrated drape, lubricate the catheter, hold the penis with the nondominant hand, clean the penis with the dominant hand, coil the catheter in the dominant hand, ask the patient to bear down, and advance the catheter into the penis.

You are assigned to care for a patient who has just undergone a mastectomy for a malignant tumor. You would most appropriately classify this self-concept component as A. Identity stressor. B. Sexuality stressor. C. Body image stressor. D. Role performance stressor.

C. Body image stressor.

A provider is discharging a client who has a prescription for home oxygen therapy via nasal cannula. Client and family teaching by the nurse should include which of the following instructions? (Select all that apply.) A. Apply petroleum jelly around and inside the nares. B. Remove the nasal cannula during mealtimes. C. Check the position of the cannula frequently. D. Report any nausea or difficulty breathing. E. Post "No Smoking" signs in prominent locations.

C. Check the position of the cannula frequently. D. Report any nausea or difficulty breathing. E. Post "No Smoking" signs in prominent locations.

1. Cyrus finds Mr. Burke restless, agitated, and confused. His pulse is 102 beats/min, and respirations are 42 breaths/min and shallow. He is sitting up in bed grasping the side rails and trying to catch his breath. He is most likely experiencing which of the following conditions? A. Hyperventilation B. Hypoventilation C. Hypoxia D. Dysrhythmia

C. Hypoxia

Margaret is a first-semester nursing student who is doing her clinical rotation in an assisted-living care facility. She is assigned to Mr. Alfred Smith, an 81-year-old African-American male who has dementia. Mr. Smith's family could no longer take care of him at home because he became combative in the evenings as a result of the effects of sundowner's syndrome. To preserve his safety, Mr. Smith now lives on the locked unit at the assisted-living facility where one or more members of his immediate and extended family come to visit every day. In addition, Mr. Smith's great niece, Harriet, works at the same facility and checks on Mr. Smith during each of her shifts. 1. Because of Mr. Smith's progressing dementia, he has difficulty feeding himself. When Margaret prompts him to eat his oatmeal at breakfast, he just stares at his spoon. Margaret picks up his spoon and wraps his hand around the handle. For which psychomotor learning skill is Margaret trying to retrain Mr. Smith? A. Set B. Mechanism C. Perception D. Guided response

C. Perception

Sally has decided to set aside 30 minutes a day to walk after work next week. Sally is in what stage of risk factor modification? A. Precontemplation B. Contemplation C. Preparation D. Action E. Maintenance

C. Preparation

1. Before Jay delegates tasks, he must make sure that he is delegating in accordance with a registered nurse's legal scope of practice. Rank in order the five rights of delegation Jay must follow. A. Right supervision/evaluation B. Right person C. Right task D. Right circumstances E. Right direction/communication

C. Right task D. Right circumstances B. Right person E. Right direction/communication A. Right supervision/evaluation

According to Maslow's hierarchy of needs, which of these needs would the patient seek to meet first? A. Self-actualization B. Self-esteem C. Shelter D. Love and belonging

C. Shelter

When completing the nursing data on a client, to complete the admission and develop a plan of care, the nurse will need to: A. test the family unit's ability to cope. B. evaluate communication patterns C. identify family unit form and attitudes D. gather health data from all family members.

C. identify family unit form and attitudes

A nurse educator is conducting a parenting class for new guardians of infants. Which of the following statements made by a participant indicates understanding? A) "I will set my water heater at 130 F." B) "Once my baby can sit up, they should be safe in the bathtub." C) "I will place my baby on their stomach to sleep." D) "Once my infant starts to push up, I will remove the mobile from over the crib."

D) "Once my infant starts to push up, I will remove the mobile from over the crib." The guardian should plan to remove crib toys (mobiles) from over the bed as soon as the infant begins to push up so the infant is unable to touch them.

A nurse is providing education for a client who has severe hypomagnesemia and is prescribed oral magnesium sulfate. Which of the following information should the nurse include in the teaching? A) "Avoid green, leafy vegetables while taking this medication." B) " You should receive a prescription fir a thiazide diuretic to take with the magnesium." C) "You should eliminate whole grains from your diet until magnesium level increase." D) "Report diarrhea while taking this medication."

D) "Report diarrhea while taking this medication." Instruct the client to report diarrhea while taking oral magnesium replacement. This is a potential adverse effect of taking oral magnesium, which could worsen the client's hypomagnesemia.

A home health nurse is discussing the dangers of carbon monoxide poisoning with a client. Which of the following information should the nurse include? A) Carbon monoxide has a distinct odor. B) Water heaters should be inspected every 5 years C) The lungs are damaged from carbon monoxide inhalation. D) Carbon monoxide binds with hemoglobin in the body.

D) Carbon monoxide binds with hemoglobin in the body. Warn the client that carbon monoxide is very dangerous because it binds with hemoglobin and ultimately reduces oxygen supplied to the tissues in the body.

The body's fluid and electrolyte balance is maintained partially by hormonal regulation. Which of the following statements shows an understanding of this mechanism? A. "The pituitary secretes aldosterone." B. "The kidneys secrete antidiuretic hormone." C. "The adrenal cortex secretes antidiuretic hormone." D. "The pituitary gland secretes antidiuretic hormone."

D. "The pituitary gland secretes antidiuretic hormone."

Health disparities are unequal burdens of disease morbidity and mortality rates by racial and ethic groups. These disparities are often exacerbated by: A. bias B. stereotyping C. prejudice D. all of the above

D. All of the above

2. Franz learns the difference between Chvostek's and Trousseau's signs. A positive Chvostek's sign elicits which of the following? A. Bilateral muscle weakness in the quadriceps B. Bilateral muscle weakness of the respiratory muscles C. Carpal spasm with hypoxia D. Contraction of facial muscles when a facial nerve is tapped

D. Contraction of facial muscles when a facial nerve is tapped

1. Jeffrey asks Mrs. Lasky about her transition to American culture. Mrs. Lasky assures Jeffrey that she is now bicultural. Which of the following statements clarifies the term biculturalism? A. Mrs. Lasky does not identify with America or Poland. B. Mrs. Lasky identifies with America more than Poland. C. Mrs. Lasky identifies with Poland more than America. D. Mrs. Lasky identifies with both America and Poland equally.

D. Mrs. Lasky identifies with both America and Poland equally.

A patient newly diagnosed with diabetes needs to learn how to use a glucometer. use of a glucometer constitutes: A. Affective learning B. Cognitive learning C. Motivational learning D. Psychomotor learning

D. Psychomotor learning

A health care provider may suspect that a patient is experiencing urinary retention when the patient has: A. large amounts of voided cloudy urine B. pain in the suprapubic region C. spasms and difficulty during urination D. small amounts of urine voided two to three times per hour.

D. small amounts of urine voided two to three times per hour.

A nurse is reviewing with a group of newly licensed nurses how to perform postural drainage. Use the ATI Active Learning Template: Nursing Skill to complete this item: DESCRIPTION OF SKILL: List the specific positions that facilitate secretion drainage from at least eight specific lung areas.

DESCRIPTION OF SKILL *Both lobes in general: high Fowler's *Apical segments of both lobes: sitting on the side of the bed *Right upper lobe, anterior segment: supine with head elevation *Right upper lobe, posterior segment: on the left side with a pillow under the right side of the chest *Right middle lobe, anterior segment: three-quarters supine with dependent in Trendelenburg *Right middle lobe, posterior segment: prone with thorax and abdomen elevation *Right lower lobe, lateral segment: on the left side in Trendelenburg *Left upper lobe, anterior segment: supine with head elevation *Left upper lobe, posterior segment: on the right side with a pillow under the left side of the chest *Left lower lobe, lateral segment: on the right side in Trendelenburg *Both lower lobes, anterior segments: supine in Trendelenburg *Both lower lobes, posterior segments: prone in Trendelenburg

metabolic acidosis causes

DKA, severe diarrhea, renal failure, shock

1. Mr. Burke's condition is causing the clinical sign of shortness of breath. Shortness of breath is referred to as _______________.

Dyspnea

1. Mr. Burke coughs up bloody sputum that Cyrus sends to the laboratory. Bloody sputum is referred to as ____________.

Hemoptysis

respiratory alkalosis s/s

Hyperventilation -lethargy, lighteheadedness, confusion -Tachycardia, Dysrhythmias r/t hypokalemia, numbness and tingling of the extremities long deep breaths

A nurse is assessing the risk factors for coronary artery disease (CAD) in a female patient. For each assessment finding (see below), indicate with an X whether the finding is modifiable, nonmodifiable or unrelated to the risk for CAD.

Nonmodifiable risk factors for CAD (e.g., age) cannot be changed. - Patient is 67 year old Modifiable risk factors for CAD include those that can be changed (e.g., hypertension, dietary choices high in fat and sodium, and a sedentary lifestyle). - B/P 146/88 mm Hg - Eats at fast-food restaurants every day for lunch - Does not participate in regular activity Unrelated -Father had type 2 diabetes mellitus -Patient treated with radiation to the chest 30 years ago to treat Hodgkin's disease

1. Ms. Ruiz is considered a vulnerable patient. Vulnerable populations are groups of patients who are more likely to develop health problems as a result of excess health risks, who are limited in access to health care services, or who depend on _______ for care.

Other

Many older homes in a neighborhood are undergoing a lot of restoration. Lead paint was used to paint the homes when they were built. The community clinic in the neighborhood is initiating a lead screening program. This activity is based on which social determinant of health?

Physical environment. This social determinant of health describes where a person lives and how that environment affects the person's health.

A nurse desires to communicate with a young woman who is Serbian and who has limited experience with being in a hospital. The nurse has 10 years of experience caring for Serbian women. The patient was admitted for a serious pregnancy complication. Apply the LEARN model and match the nurse's behaviors with each step of the model. ____ 1. L ____ 2. E ____ 3. A ____ 4. R ____ 5. N a. The nurse notes that she has learned that fathers can visit mothers at any time in both Serbia and the United States. b. The nurse shares her perception of the woman's experiences as a patient. c. The nurse asks the patient how she can maintain bed rest when she returns home d. The nurse attends to the patient and listens to her story about hospitals in Serbia. e. The nurse involves the patient in a discussion of the treatment options for her condition.

____ 1. L d. The nurse attends to the patient and listens to her story about hospitals in Serbia. ____ 2. E b. The nurse shares her perception of the woman's experiences as a patient. ____ 3. A a. The nurse notes that she has learned that fathers can visit mothers at any time in both Serbia and the United States. ____ 4. R c. The nurse asks the patient how she can maintain bed rest when she returns home ____ 5. N e. The nurse involves the patient in a discussion of the treatment options for her condition. Listen with empathy and understanding to the patient's perception of the problem; Explain your perceptions of the problem (physiological, psychological, spiritual, and/or cultural); Acknowledge and discuss cultural differences and similarities between you and your patient; Recommend treatment (involving the patient); and Negotiate agreement (incorporate selected aspects of the patient's culture into patient-centered care).

A patient complains of chest pain. When assessing the pain, you decide that its origin is cardiac - rather than respiratory or gastrointestinal- when it: a. does not occur with respiratory variations. b. is peripheral and may radiate to the scapular regions. c. is aggravated by inspiratory movements. D. is nonradiating and occurs during inspiration

a. does not occur with respiratory variations.

A patient with a tracheostomy has thick tenacious secretions. To maintain the airway, the most appropriate action for the nurse includes: a. tracheal suctioning. b. oropharyngeal suctioning c. nasotracheal suctioning d. orotracheal suctioning

a. tracheal suctioning.

1. Ms. Ruiz tells Michelle that she left home at age 14 and was homeless until she met Roberto, who found her a job and provided shelter. When Ms. Ruiz was a homeless teenager, she was at an increased risk for which of the following because of her immaturity and lack of a nuclear family? A. Illness B. Risky behaviors C. Compromised self-esteem D. Malnutrition

b. Risky behaviors

Trevor is a nursing student who works at a local clinic as a medical assistant three times a week while he is in nursing school. He has been a medical assistant for 6 years and decided to attend nursing school this semester to expand his career options. Trevor kept his medical assisting job in hopes that gaining the "hands on" experience would make nursing school more manageable; plus he needed the income to help offset the cost of nursing school. At the clinic Trevor cares for a wide range of patients from newborns to the older adult. Today the clinic is hosting a "Love Your Child" health drive and is providing wellness examinations and immunizations for children. 1. Trevor is assigned to the immunization station at the health drive where he is responsible for administering vaccines to the children. Immunizations are an example of _________________ and _____________ prevention.

primary and illness

metabolic alkalosis causes

severe vomiting, excessive GI suctioning, diuretics, excessive NaHCO3, overuse antacids, ingestion backing soda


Set pelajaran terkait

Functional Anatomy Exam 2 Material

View Set

final exam review - intro to business

View Set

Macroeconomics Test #3 Chapter 31

View Set

Chapter 2: descriptive statistics

View Set

Business Law Sole Proprietors, Partnerships, Corporations, LLC

View Set

Introduction To Polynomials Assignment

View Set